Anda di halaman 1dari 59

alternatifa

UTUL UGM BAHASA INDONESIA 2008 481


1. Kehidupan orang Malind, salah satu suku Serikat akan mempengaruhi proyeksi
di Papua yang masih mengandalkan perekonomian di negara-negara Asia,
perburuan dan penokokan sagu untuk terutama Indonesia. Ekonomi global
mencari makanan, sangat dipengaruhi diperkirakan mengalami penurunan
alam. Selama ribuan tahun kekayaan pertumbuhan sebesar 0,4%, dari 5,2%
hutan ulayat Malind membuat hidup pada tahun 2007 menjadi 4,8% pada
mereka berkelimpahan. Sagu di mana- tahun 2008. Negara-negara di Asia
mana, saham (kanguru kecil) dan rusa Tenggara diperkirakan mengalami
berlarian, burung-burung berkicau di tekanan paling parah akibat
sekitar kampung adat, dan ikan pun melambatnya ekonomi yang terjadi di
melimpah. Namun, sekitar dua puluh Amerika.
tahun yang lalu, sebuah sentakan budaya
Berdasarkan bacaan di atas, pernyataan
terjadi ketika lebih dari 48.000 hektare
berikut ini yang salah adalah …
hutan ulayat dibuka menjadi sawah bagi
para transmigran. A. Penurunan pertumbuhan ekonomi
di Amerika Serikat berdampak pada
Dari bacaan di atas dapat disimpulkan
penurunan pertumbuhan ekonomi
bahwa …
global.
A. Kehidupan suku Malind telah B. Pada tahun 2008 pertumbuhan
berubah dari sistem meramu dan ekonomi Amerika Serikat
berburu ke bercocok tanam. diperkirakan lebih rendah daripada
B. Pembukaan hutan ulayat menjadi pertumbuhan ekonomi global.
sawah telah mempengaruhi C. Pada tahun 2008 Amerika Serikat
kehidupan suku Malind. diperkirakan akan mengalami
C. Kesejahteraan suku Malind penurunan pertumbuhan ekonomi
semakin meningkat sejak sebesar 1%.
dibukanya hutan ulayat menjadi D. Sebagai dampak penurunan
sawah. pertumbuhan ekonomi Amerika
D. Sejak dibukanya hutan ulayat Serikat pertumbuhan ekonomi di
menjadi sawah, suku Malind tidak Asia tidak akan turun lebih besar
lagi dapat berburu. daripada besarnya angka
E. Suku Malind belajar bercocok penurunan pertumbuhan ekonomi
tanam di sawah dari para Amerika Serikat.
transmigran yang datang ke daerah E. Jika pertumbuhan ekonomi
itu. Amerika Serikat turun 2%, sebagai
dampaknya pertumbuhan ekonomi
2. Pada tahun 2008 pertumbuhan ekonomi negara Asia akan turun sebesar 1%-
Amerika Serikat diperkirakan melambat 2%.
menjadi 2% atau sedikit menurun
dibandingkan dengan tahun sebelumnya. 3. Satu pelajaran penting yang dapat
Menurut IMF, sekitar 1% penurunan diambil dari pelbagai macam upaya
pertumbuhan ekonomi di Amerika Serikat penyelesaian konflik Palestina dan Israel
akan menurunkan pertumbuhan ekonomi adalah bahwa konferensi atau
di Asia sekitar 0,5%-1%. Dampak resesi perundingan apa pun tidak mampu
global yang berasal dari resesi di Amerika mengubah warna jalan-jalan di Timur
alternatifa

Tengah. Tatkala perdamaian menderu begitu saja tanpa menyiram atau


kencang di ruang konferensi, suara membersihkan peturasannya.
tembakan dan kekerasan acap kali
Topik paragraf di atas adalah…
mewarnai jalan-jalan di kawasan itu.
Persoalan konflik Palestina dan Israel A. Persoalan toilet umum.
bukan persoalan elite belaka. Persoalan B. Budaya masyarakat Indonesia
ini telah menyebar dan meluas menjadi C. Organisme dalam tanah.
persoalan masyarakat Timur Tengah D. Kebiasaan lama orang dalam buang
secara umum. hajat.
E. Pengunjung toilet umum.
Berdasarkan isi bacaan di atas,
pernyataan berikut ini benar, kecuali…
5. Investasi perlu didorong ke luar Jawa.
A. Konflik Palestina dan Israel bukan Selain kekayaan sumber daya alamnya,
hanya persoalan kedua bangsa itu. investasi di luar Jawa juga lebih menarik
B. Dalam penyelesaian konflik karena daya beli rata-rata masyarakat di
Palestina dan Israel antara hasil luar Jawa sebenarnya lebih tinggi
perundingan dengan kenyataan di dibandingkan dengan daya beli rata-rata
lapangan sering tidak sejalan. penduduk Pulau Jawa. Namun,
C. Sampai saat ini belum ada keterbatasan infrastruktur di luar Pulau
perundingan yang dapat Jawa masih menjadi penghambat
menyelesaikan konflik Palestina pengembangan investasi di sejumlah
dan Israel. sektor.
D. Hanya perundingan para elite
Dalam bacaan di atas tersirat bahwa …
kedua negara yang dapat
menyelesaikan konflik Palestina A. Selama ini investasi lebih terpusat
dan Israel. di Pulau Jawa
E. Masyarakat di Timur Tengah belum B. Belum ada investasi di luar Pulau
memiliki pandangan yang sama Jawa
dalam menyelesaikan konflik C. Investasi di Pulau Jawa sudah tidak
Palestina dan Israel. menguntungkan lagi.
D. Investasi di luar Pulau Jawa akan
4. Selain masih minim dan kurang mendatangkan keuntungan besar.
terurusnya fasilitas, persoalan toilet E. Investasi di luar Pulau Jawa
umum yang kotor di kota berakar pada terhambat oleh keterbatasan
masalah budaya masyarakat. Dalam infrastruktur.
budaya masyarakat Indonesia yang
agraris, masyarakat pada zaman dahulu 6. (1) Jumlah ini meningkat tajam dari
pernah punya kebiasaan buang hajat di tahun sebelumnya yang hanya tiga kasus.
alam terbuka, seperti sungai, empang, (2) Menurut Direktur LBH Palembang, Eli
hutan, atau semak belukar di belakang Gustina, pada tahun 2007 terjadi 39 kasus
rumah, tanpa harus membersihkannya. penyerobotan tanah milik warga di
Kotoran dibiarkan saja karena bisa Provinsi Sumatera Selatan. (3) Demikian
langsung diurai organisme dalam tanah, salah satu kesimpulan refleksi akhir
tumbuhan, atau air. Di kota orang harus tahun LBH Palembang. (4) Konflik ini di
buang hajat di toilet, tetapi kebiasaan antaranya terjadi di Kabupaten Musi
lama masih dibawa. Buktinya, masih Banyuasin dan Ogan Komering Ilir. (5)
banyak pengunjung toilet umum pergi Lembaga Bantuan Hukum Palembang
alternatifa

mencatat peningkatan ketegangan Kalimat yang tepat untuk melengkapi


akibat kasus penyerobotan tanah milik titik-titik dalam bacaan di atas adalah …
warga oleh sejumlah perusahaan
A. Para peternak itik di wilayah itu
perkebunan tahun 2007.
membuat penemuan baru.
Untuk menjadi sebuah paragraf yang B. Demikianlah yang terjadi pada para
runtut, kalimat-kalimat di atas harus peternak itik di wilayah itu
disusun menjadi … C. Dua jenis itik disilangkan oleh para
peternak itik di wilayah itu untuk
A. 1- 2 – 4 – 3 - 5
mendapatkan varietas itik baru.
B. 2 - 4 - 3 - 1- 5
D. Di wilayah itu para peternak itik
C. 5-1-3-4-2
menunjukkan kreativitasnya.
D. 2-3-4-1-5
E. Itulah sebabnya para peternak itik
E. 5 - 4 - 3 - 2- 1
di wilayah itu menunjukkan
kreativitasnya.
7. Asam folat selama ini dikenal luas sebagai
suplemen wajib pada masa pra kehamilan
9. Kata yang mengalami perubahan makna
dan kehamilan (...) timbul anggapan
karena asosiasi terdapat dalam kalimat
bahwa zat ini hanya dibutuhkan oleh

para calon ibu. (...), baik pria maupun
A. Persentase sarjana yang belum
wanita sama-sama membutuhkan
mendapatkan pekerjaan semakin
asupan harian asam folat (…), kebutuhan
bertambah.
harian asam folat pria dewasa sama
B. Para pegiat perempuan semakin
besarnya dengan kebutuhan wanita
berani memperjuangkan hak-
dewasa.
haknya.
Kata yang tepat untuk mengisi tiga C. Karena kesantunan nya, oleh
bagian kosong dalam bacaan di atas temannya dia disebut sebagai
berturut-turut adalah… pendeta.
D. Sesekali dia melontarkan kritikan
A. Lalu, oleh karena itu, sedangkan. pedas terhadap kebijakan
B. Dan, walaupun, adapun. atasannya yang dianggapnya keliru.
C. Sehingga, padahal, bahkan. E. Pada waktu melaksanakan tugas
D. Meskipun, akan tetapi, jadi. jurnalistiknya, dia selalu berusaha
E. Karena, sebenarnya, dengan menolak pemberian amplop dalam
demikian. setiap peristiwa yang diliputnya.

8. Terbatasnya lahan dan modal usaha di 10. Untuk membangun tanggul penahan
tengah semakin tingginya tuntutan banjir, pemerintah akan merelokasi
kebutuhan hidup acap kali justru warga yang tinggal di bantaran sungai.
mendorong kreativitas. Tidak pernah
puas dengan hasil kerja yang sama Yang dimaksud dengan bantaran dalam
bertahun-tahun, terobosan-terobosan kalimat di atas adalah …
baru pun dilakukan untuk memperoleh
A. Bagian hilir.
hasil yang lebih baik. .... Mereka berhasil
B. Bagian hulu.
mengembangkan budi daya ternak itik
C. Bagian sisi kanan dan kiri.
unggul organik yang merupakan
D. Bagian yang tidak dialiri air.
persilangan dari itik alabio Kalimantan
E. Bagian lereng tebing.
Timur dengan itik Mojosari Jawa Timur.
alternatifa

11. Kalimat Dalam pergaulan sehari-hari D. Dibutuhkan aturan yang jelas untuk
sering terlihat ada orang yang tidak melaksanakan pilkada ulang.
dapat menerima saran orang lain E. Indonesia kehilangan julukannya
memiliki pola yang sama dengan sebagai zamrud khatulistiwa.
kalimat…
A. Dengan terbata-bata dikisahkannya 14. Kalimat yang baku adalah …
peristiwa menyedihkan yang A. Untuk mengatasi banjir Bengawan
menimpa dirinya itu. Solo, penanaman pohon di sekitar
B. Di sekitar kita ada orang yang Waduk Gajah Mungkur dan
mendorong dirinya untuk membaca sepanjang bengawan tidak dapat
evaluasi dan menghadiahi dirinya ditawar lagi.
sendiri jika hasilnya bagus. B. Sedangkan penanganan jangka
C. Karena hampir setiap individu pendek dengan memperbaiki
berinisiatif untuk berkembang dan tanggul sepanjang bengawan yang
berubah, tidak perlu rusak.
dikumandangkan gerakan C. Juga dengan membuat bendungan
perubahan. kecil di puluhan titik sepanjang
D. Agar dapat terus bekerja di bengawan.
perusahaan itu, diikutinya berbagai D. Di sepanjang bengawan
kursus keterampilan. menemukan lebih dari 20 tempat
E. Dalam lingkungan yang terbatas yang potensial untuk dibangun
disampaikannya pesan yang agak waduk.
rahasia itu. E. Jika di tempat tersebut dibangun
waduk kecil-kecil, akan bermanfaat
12. Tidak dapat disangkal bahwa mengurangi banjir.
kebanyakan orang pada saat ini lebih
memilih menggunakan kamera yang ada 15. Kalimat Tugas utama pemerintah
pada telepon genggam daripada secara adalah memperhatikan kepentingan
khusus membeli kamera biasa karena masyarakat kelas bawah mempunyai
kamera analog dianggap kurang praktis. pola yang sama dengan kalimat …
A. Tingginya angka inflasi akan
Inti kalimat tersebut adalah …
berpengaruh terhadap masyarakat
A. Disangkal. kelas bawah.
B. Orang memilih kamera telepon B. Oligarki (kekuasaan di tangan
genggam. segelintir orang) dan plutokrasi
C. Orang menggunakan kamera. (pemerintahan oleh sekelompok
D. Orang membeli kamera. orang kaya) menjadi penumpang
E. Kamera analog tidak praktis. gelap sistem demokrasi
C. Tidak satu pun partai politik di
13. Kalimat berikut baku, kecuali … Indonesia terbebas dari sindrom
A. Pesan terpenting dari sejarah oligarki dan plutokrasi.
adalah pesan tentang kesalahan. D. Sistem kepartaian yang sederhana
B. Pesan itu kami belum terima dapat mengerem pertumbuhan
sampai sekarang. oligarki dan plutokrasi.
C. Media massa ramai mengulas E. Partai politik dapat menempatkan
temuan rekening liar. kadernya untuk mengisi jabatan
alternatifa

publik sesuai dengan kapasitas dan pencaharian baru warga Pulau


kapabilitasnya. Komodo.

16. Kalimat yang tidak baku adalah… 18. Untuk kurangi pemanasan global maka
A. Sesuai dengan kesepakatan, masyarakat diminta untuk meminimalisir
pertunjukan itu akan diadakan di pemakaian kendaraan bermotor.
Jawa dan Sumatra.
Kalimat di atas akan menjadi kalimat
B. Masa yang akan datang kenaikan
baku apabila diperbaiki dengan cara
yang tidak terkendali dari harga
berikut, kecuali…
kebutuhan sehari-hari tidak boleh
terjadi lagi. A. Mengganti kata untuk pada awal
C. Ramalan BMG semakin kita kalimat dengan dalam.
perhatikan, terutama setelah B. Mengubah kata kurangi menjadi
muncul berbagai dampak tak mengurangi.
terduga dari perubahan iklim. C. Menambahkan tanda koma (,)
D. Di negara-negara bermusim empat, sesudah kata global.
ramalan cuaca dan geofisika sudah D. Menghilangkan kata maka.
lama menjadi bagian dari E. Mengubah kata meminimalisir
kehidupan sehari-hari. menjadi meminimalkan.
E. Untuk keperluan pelestarian
lingkungan diperlukan tanggung 19. Kalimat yang penulisannya mengikuti
jawab kolektif dan antisipatif dari EYD adalah …
para pengusaha yang bergerak di A. Sepuluh tahun yang lalu Keluarga
bidang pengusahaan hutan. Satrio tinggal di jalan RE
Martadinata, No.5.
17. Kalimat Kegiatan ekoturisme untuk B. Sebagian besar dari kita
mengkonservasi alam sering tidak beranggapan bahwa kebudayaan
sejalan dengan kebiasaan masyarakat Timur lebih halus jika
setempat memiliki pola yang sama dibandingkan dengan kebudayaan
dengan kalimat … Barat.
A. Banyak pegiat ekoturisme mengakui C. la mengakui ke dalaman wawasan
bahwa kesalahan yang sering terjadi tokoh masyarakat kampung Naga
dalam mengembangkan ekoturisme itu.
adalah pengemasan yang semata- D. Pengusaha besar itu menghadiahi
mata mengikuti selera masyarakat. atlit-atlit berprestasi internasional
B. Pembuatan patung kayu komodo dengan bea siswa X untuk belajar
pasti akan menghabiskan pepohonan keluar negeri.
yang sekarang sudah jarang. E. Beberapa tokoh anti rasisme dari
C. Larangan pengambilan kayu waru beberapa negara sepakat untuk
laut di Pulau Komodo menanda tangani suatu
dilatarbelakangi oleh terbatasnya kesepahaman.
populasi vegetasi di sana.
D. Haji Zainun terus membuat patung 20. Penurunan cukai impor beras menjadi
komodo di sela-sela kesibukannya Rp.450,- per kilogram per 1 Januari 2008
sebagai nelayan. dimaksudkan agar Bulog dapat
E. Keterampilan membuat patung mengendalikan harga beras
komodo merupakan sumber mata
alternatifa

Kalimat tersebut akan benar ejaannya


apabila…

A. Rp.450,- ditulis Rp450,00.


B. Per kilogram ditulis perkilogram.
C. Per 1 Januari 2008 ditulis per-1
Januari 2008.
D. Impor ditulis import.
E. Sebelum kata agar ditambahkan
tanda koma (.)
alternatifa

UTUL UGM BAHASA INDONESIA 2009 931

Bacalah bacaan berikut dengan cermat, kemudian jawablah pertanyaan nomor 1 sampai dengan
3.

Begitu matahari terbit di Batu Kambar, Desa Hinas Kiri, Kecamatan Batang Alai Timur,
Kabupaten Hulu Sungai Tengah, Kalimantan Selatan, ratusan warga keluar rumah. Ada yang langsung
pergi ke pahumaan (ladang), ada yang pergi ke pasar, ada pula yang berangkat ke sekolah. Warga di
pedalaman Pegunungan Meratus memang terbiasa bangun pagi karena mereka juga sudah terbiasa
tidur sore akibat tidak adanya aliran listrik. Karena itu, aktivitas pagi di Pegunungan Meratus betul-
betul menggeliat dan dinamis. Hari itu kebetulan hari pasar tradisional yang hanya ada sekali dalam
sepekan. Masyarakat dari berbagai balai adat Dayak Meratus, yang jarak tempuhnya hingga 18 jam
dengan jalan kaki, berduyun-duyun ke pasar untuk membeli keperluan hidup “modern”, mulai dari
bahan pangan, sandang, hingga papan.

Tradisionalisme, mulai dari cara makan, cara mandi, cara berinteraksi, hingga pandangan
hidup, betul-betul masih melekat dan menjadi identitas warga pedalaman Pegunungan Meratus. Lambat
laun keterisolasian pedalaman memang bisa ditembus, tetapi hingga kini kehidupan alami masih
melekat. Baik di hulu Sungai Selatan maupun di hulu Sungai Tengah yang mempunyai komunitas adat
terbesar di Kalimantan Selatan suasananya sama-sama masih tradisional. Tradisionalisme di satu sisi
menjadi kebanggaan, tetapi di sisi lain menyisakan pertanyaan akan capaian sebuah kesejahteraan
masyarakat.

1. Judul yang tepat untuk bacaan di atas adalah ….


A. Kisah Kehidupan dari Batu Kambar, Desa Hinas Kiri.
B. Kehidupan Tradisional Masyarakat Pegunungan Meratus.
C. Identitas Warga Pedalaman Pegunungan Meratus.
D. Tradisionalisme Menjadi Kebanggaan bagi Masyarakat Pegunungan Meratus.
E. Selama 18 jam Berjalan Menuju Pasar untuk Keperluan Hidup "Modern"

2. Inti kalimat keenam dalam paragraf pertama bacaan di atas adalah ….


A. Masyarakat berjalan ke pasar selama 18 jam. "
B. Masyarakat membeli keperluan hidup "modern".
C. Masyarakat berduyun-duyun ke pasar.
D. Masyarakat Dayak Meratus terdiri atas berbagai balai adat.
E. Masyarakat Dayak Meratus berjalan kaki ke pasar.

3. Dalam bacaan di atas terdapat pernyataan implisit berikut, kecuali ….


A. Warga Pedalaman Pegunungan Meratus tidak mengenal kendaraan bermotor.
B. Warga Pedalaman Pegunungan Meratus belum menggunakan listrik.
C. Tradisionalisme di satu sisi adalah kesedihan atas keterbelakangan
D. Pasar tradisional hanya ada satu kali dalam seminggu.
E. Masyarakat Pegunungan Meratus belum menikmati kebutuhan sekunder dan tersier.

4. Pada saat ini, kerisauan ternyata bukan hanya milik para petani yang kesulitan mendapatkan
pupuk. Bambang Riyanto, seorang perajin jenang dan tempe keripik asal Purwokerto, bahkan
harus pontang-panting mencari minyak tanah agar usahanya itu tetap berlangsung. Kerisauan

1
alternatifa

jelas membayang ketika akan memasuki musim liburan akhir tahun lalu. "Musim panen" bagi
industri kecil makanan oleh-oleh ini tentu tidak akan dibiarkan lewat begitu saja. Untuk
mendapatkan minyak tanah yang cukup untuk produksi, ia harus mengerahkan seluruh anggota
keluarganya, bahkan para tetangganya, untuk ikut antre membeli minyak tanah.

Pikiran pokok paragraf di atas adalah….

A. Kerisauan para petani dan perajin makanan oleh--oleh di Purwokerto karena sulitnya
pupuk dan minyak tanah.
B. "Musim panen" bagi industri kecil makanan oleh- oleh.
C. Kerisauan Bambang Riyanto, seorang perajin jenang dan tempe keripik asal Purwokerto.
D. Kelangkaan minyak tanah di Purwokerto.
E. Suka duka perajin jenang dan tempe keripik asal Purwokerto.

5. Jangan dikira hanya Italia yang mempunyai menara miring, Indonesia pun mempunyai bangunan
bersejarah "berbau miring". Istana Tampaksiring, Bali, memang tidak miring seperti Pisa, tetapi
letaknya di desa yang sejak zaman baheula melegenda sebagai tanah miring. Kata tampaksiring
dicomot dari bahasa setempat, tampak dan siring yang artinya "telapak miring". Pembangunan
istana Tampaksiring tak bisa dipisahkan dari kiprah mantan Presiden RI Bung Karno (BK).
Konon, pemilihan tempat, proses pembangunan, hingga finishing Istana Tampak Siring, semua
melibatkan BK. la sudah mengenal lahan berpemandangan indah ini jauh sebelum disulap
menjadi istana. Sebelum 1955 (istana dibangun 1957) BK sudah sering istirahat di bekas tempat
tetirah Raja Gianyar yang dikeramatkan penduduk setempat.

Inti paragraf di atas adalah …

A. lstana Tampaksiring yang merupakan bekas tempat tetirah Raja Gianyar sering digunakan
oleh BK untuk istirahat.
B. Seperti Italia, Indonesia juga memiliki bangunan yang didirikan di atas tanah berkontur
miring.
C. lstana Tampaksiring dibangun di desa bertanah miring sehingga bangunan ini juga "berbau
miring".
D. Raja Gianyar membangun lstana Tampak Siring pada tahun 1957 dengan melibatkan BK.
E. lstana Tampaksiring yang dibangun di desa bertanah miring dan berpemandangan indah
tidak bisa dipisahkan dari BK.

6. Adat Badui yang ketat tercermin dalam pepatah "Yang panjang tidak boleh dipotong, yang pendek
tidak boleh ditambah". Hal ini sengaja ditekankan agar tradisi lama yang sesungguhnya tetap
dapat dipertahankan. Namun, tidak berarti masyarakat Badui tidak menerima sesuatu yang baru,
atau sama sekali menutup sesuatu dari dunia luar. Di sana sini terlihat orang Badui membuat
pelbagai percobaan dengan mengubah kombinasi atau variasi unsur-unsur adat walaupun tetap
dalam batas-batas yang ditetapkan oleh tetua adat.

Inti paragraf di atas adalah….

A. Adat Badui menekankan kemurnian tradisi lama sehingga sedapat mungkin mereka
menolak berkontak dengan dunia luar.
B. Adat Badui berkembang dengan inovasi unsur--unsur adat dan membuka diri terhadap
campur tangan dari dunia luar.

2
alternatifa

C. Adat Badui mempertahankan tradisi dan mengembangkannya dengan pelbagai percobaan


atas variasi unsur-unsur adat.
D. Adat Badui telah mengubah tradisi lama dengan tradisi baru sehingga menjadi lebih maju
dan modern.
E. Adat Badui mengikuti batas-batas yang ditetapkan oleh tetua adat yang menekankan
kesederhanaan.

7. Beberapa bulan lalu Dinas Kepurbakalaan, Budaya, dan Pariwisata Jambi memprakarsai napak
tilas Ekspedisi Pamalayu. Ekspedisi penyusuran Sungai Batanghari ini bertujuan mencatat segala
potensi yang ada di sepanjang rute untuk perencanaan dan pengembangan pariwisata daerah
tersebut. Penyusuran itu mengingatkan kita pada Ekspedisi Pamalayu pada tahun 1286, yakni
ekspedisi diplomatik Kerajaan Singasari ke Kerajaan Melayu. Ekspedisi atas prakarsa Raja
Kertanegara itu disertai pengiriman arca Amoghapasa. Menurut catatan sejarah, pengiriman area
itu menempuh rute Laut Jawa, laut bagian timur Pulau Sumatera, hingga memasuki muara dan
hulu Sungai Batanghari.

Dari bacaan di atas dapat disimpulkan bahwa ….

A. Pengiriman arca Amoghapasa ke Kerajaan Melayu menempuh perjalanan jauh


B. Penyusuran itu untuk mengenang kebesaran Ekspedisi Pamalayu pada tahun 1286, yakni
ekspedisi diplomatik Kerajaan Singasari ke Kerajaan Melayu
C. Ekspedisi Pamalayu tahun 1286 menunjukkan bahwa Kerajaan Singasari memiliki kekuatan
maritim yang kuat.
D. Napak tilas Ekspedisi Pamalayu itu dilakukan dalam rangka mengembangkan pariwisata
sepanjang rule pelayaran yang dilewati utusan Raja Kertanegara ke Kerajaan Melayu.
E. Napak tilas Ekspedisi Pamalayu itu dibiayai oleh Dinas Kepurbakalaan, Budaya, dan
Pariwisata Jambi

8. Upacara wiwit termasuk salah satu tradisi penting dalam dunia pertanian masyarakat Jawa,
khususnya dunia pertanian yang berkaitan dengan budi daya tanaman pokok (padi). Hingga kini,
upacara tersebut masih bisa dijumpai di pelosok pedesaan walaupun semakin sedikit yang
melestarikannya. Upacara wiwit semakin langka dijumpai seiring dengan munculnya banyak
perubahan nilai-nilai budaya dalam kehidupan sosial komunitas petani, semakin sempitnya
lahan pertanian mereka, dan adanya perubahan sistem produksi pertanian di Jawa.

Dari bacaan di atas dapat disimpulkan bahwa ….

A. Langkanya upacara wiwit karena sistem pertanian di Jawa mulai berubah.


B. Langkanya upacara wiwit karena banyaknya perubahan nilai-nilai budaya.
C. Upacara wiwit masih bisa dijumpai di pelosok pedesaan di Jawa walaupun semakin sedikit
yang melestarikannya
D. Upacara wiwit yang kini makin langka merupakan tradisi penting dalam budidaya tanaman
padi di Jawa.
E. Upacara wiwit termasuk salah satu tradisi penting dalam dunia pertanian masyarakat Jawa.

9. Wakil Presiden Bank of China, Zhao Min, menegaskan bahwa belanja konsumsi di Amerika
Serikat, negara dengan ekonomi terbesar di dunia akan turun 10% tahun ini akibat lesunya
ekonomi global. Konsumsi Amerika dalam beberapa tahun ini diperkirakan menyusut tajam.
Penurunan belanja negeri Obama itu diprediksi akan lebih besar lagi dalam tiga atau empat tahun

3
alternatifa

mendatang. Padahal, Amerika dan Jepang merupakan pasar ekspor terbesar produk Cina.
Akibatnya, krisis di Amerika tersebut membuat pertumbuhan ekonomi Cina mulai melambat
beberapa bulan ini. lmbas krisis itu membuat banyak perusahaan di Negeri Tirai Bambu menjadi
goyah.

Berdasarkan isi bacaan di atas, pernyataan berikut ini benar, kecuali….

A. Cina banyak tergantung perdagangannya pada Amerika Serikat.


B. Penurunan belanja Amerika sebesar 10% akan diikuti lagi dengan penurunan konsumsi
negara itu di tahun-tahun berikutnya.
C. Belanja konsumsi sebesar-besarnya merupakan satu-satunya jalan untuk memperbaiki
ekonomi Amerika.
D. Banyak perusahaan di Cina menjadi tidak stabil setelah pembelian produknya berkurang di
Amerika.
E. Zhu Min berpendapat bahwa krisis di Amerika membuat pertumbuhan ekonomi Cina tidak
begitu signifikan beberapa bulan ini.

10. Dalam era globalisasi ini komputer telah dipergunakan dalam banyak bidang kehidupan,
misalnya dalam bidang kesehatan, pertahanan dan keamanan, termonuklir, perbankan,
transportasi, dan sensus penduduk. Ditinjau dari segi ukurannya, komputer yang dipergunakan
itu cukup bervariasi, mulai dari komputer mini, misalnya jam tangan digital, kalkulator dan
telepon seluler; komputer mikro, misalnya laptop dan PC (personal computer); hingga komputer
makro, misalnya server dalam unit-unit LAN (Local Area Network) dan mainframe yang biasa
dipergunakan di bank-bank atau perusahaan- perusahaan besar lainnya.

Paragraf di atas dapat diringkas menjadi ….

A. Komputer menjadi bagian penting dalam era globalisasi.


B. Komputer yang dipakai dalam kehidupan sehari- hari memiliki ukuran bervariasi.
C. Komputer digunakan dalam berbagai bentuk dan bidang kehidupan.
D. Komputer menjadi perangkat penting dalam mempermudah pekerjaan manusia.
E. Komputer digunakan di bank-bank dan perusahaan-perusahaan besar.

11. Kalimat yang efektif adalah …


A. Data perilaku manajerial yang dikumpulkan oleh peneliti dan tenaga teknisi dari lokasi
penelitian diperoleh dengan cara mewawancarai dan menyebarkan kuesioner kepada
informan.
B. Data perilaku manajerial yang dikumpulkan oleh peneliti dan dibantu tenaga teknisi dari
lokasi penelitian diperoleh dengan metode wawancara dan kuesioner kepada informan.
C. Data perilaku manajerial yang dikumpulkan oleh peneliti dengan asisten tenaga teknisi dari
lokasi penelitian diperoleh dengan metode wawancara dan kuesioner kepada informan.
D. Data perilaku manajerial yang dikumpulkan peneliti dan dibantu tenaga teknisi dengan
datang langsung ke lokasi penelitian diperoleh dengan metode wawancara dan kuesioner
kepada informan.
E. Data perilaku manajerial yang dikumpulkan oleh peneliti dan tenaga teknisi diperoleh lewat
wawancara dan kuesioner.

12. Kalimat berikut tidak baku, kecuali ….

4
alternatifa

A. Dengan adanya "liberalisasi" sistem pemilihan semakin membuat profil DPR mendatang
sulit diprediksi dan tidak mempunyai jaminan kinerja mereka semakin membaik.
B. Menurut diagnosa dokter, pasien itu positif mengidap penyakit flu burung.
C. Dalam Bab I penelitian ini membicarakan tentang latar belakang, masalah, dan landasan
teori.
D. Jadi, di negara kita tidak dikenal diskriminasi terhadap warga negara yang menyandang
cacat.
E. Untuk merenovasi museum itu memerlukan dana yang tidak sedikit.

13. Yang merupakan kalimat baku adalah ….


A. Buku ini terdapat istilah-istilah kekerabatan pada orang Jawa dan Batak dalam susunan
masyarakat yang ditulis berdasarkan ilmu antropologi.
B. Dari beberapa pokok persoalan yang diberikan untuk membandingkan dua atau lebih
dialek, antara lain dalam bidang fonetik atau semantik.
C. Adapun yang akan saya uraikan di sini ialah kebersihan dan kesehatan, saya terdorong
untuk mengemukakannya, karena sering dilalaikan orang dan sesungguhnya kebersihan
dan kesehatan itu perlu, sebab dengan bersih tentu akan menjadi sehat.
D. Karena nilai yang didapatkan lebih besar dari penolakan, maka hipotesis nihil ditolak.
E. Jika data yang diolah tidak memenuhi syarat, baik dari segi kuantitas maupun kualitasnya,
hasil penelitian di lapangan itu tidak dapat dipertanggungjawabkan secara ilmiah.

14. Kalimat berikut tidak baku, kecuali….


A. Dengan teknik tersebut para astronom dapat memperkirakan ukuran planet meskipun
belum diketahui massanya, namun diperkirakan antara 5,7 hingga 11 massa bumi.
B. Landasan pacu Bandara Ahmad Yani Semarang tergenang sepanjang 500 meter dan harus
menggunakan 23 pompa air untuk menyurutkan genangan.
C. Bandara menunda semua penerbangan sejak Minggu pagi hingga pukul 21.00 karena
kondisi landasan pacu masih buruk.
D. Komisi pemilihan umum berencana mengatur penetapan calon terpilih yang berpihak pada
calon perempuan, jika partai politik meraih tiga kursi DPR/DPRD, salah satunya mesti
diberikan kepada calon perempuan yang perolehan suaranya terbanyak.
E. Berdasarkan pada data di Dinas Bina Marga Pemkot Surabaya, maka titik-titik genangan itu
menyebar hampir di seluruh wilayah Surabaya mulai sisi paling barat, timur, hingga ke
selatan.

15. Beberapa waktu lalu berita tentang susu bermelamin, biskuit beracun, dan bahan kosmetik yang
mengandung bahan merkuri dan hidrokinon dengan cepat tersebar melalui media massa
sehingga meresahkan pedagang dan konsumen yang memiliki, menyimpan, memperjualbelikan,
atau menggunakannya.

Inti kalimat di atas adalah….

A. Berita meresahkan.
B. Berita tersebar.
C. Bahan beracun tersebar.
D. Pedagang dan konsumen resah.
E. Pedagang dan konsumen menggunakan.

5
alternatifa

16. Dalam konferensi internasional kebudayaan Melayu itu tidak diputuskan tempat
penyelenggaraan konferensi berikutnya.

Kalimat yang memiliki pola yang sama dengan pola kalimat di atas adalah ….

A. Obama, Presiden Amerika yang baru terpilih itu, mengawali karirnya dari bawah.
B. Seperti telah kita ketahui, perekonomian Amerika Serikat sangat berpengaruh.
C. Berdasarkan pengarahan pimpinan, pembangunan gedung baru itu dapat dilakukan secara
bertahap.
D. Kita sadari bahwa bencana alam dapat datang kapan saja tanpa kita duga.
E. Telah kami bicarakan dalam rapat terdahulu bahwa kenaikan gaji pegawai baru bisa
dilaksanakan bulan depan.

17. Kata jual yang bermakna literal terdapat dalam kalimat ….


A. Sejarah membuktikan bahwa sebagian pribumi menjual bangsanya sendiri pada tangan
penjajah.
B. Kau tidak akan mendapat jodoh kalau terlalu jual mahal.
C. Bagi sebagian anak muda, blog adalah media untuk jual tampang.
D. Para penipu berkedok hadiah biasanya menjual nama perusahaan tertentu.
E. Oknum paramedis di RS swasta itu terlibat aksi jual beli gigi palsu.

18. Semua pimpinan partai kini sedang memeras otak untuk memenangkan pemilu.

Gabungan kata yang semakna dengan memeras otak adalah ….

A. Penyanyi itu memamerkan suara emasnya.


B. Buruh tani itu setiap hari membanting tulang.
C. Pemerintah daerah tidak boleh hanya berpangku tangan ketika melihat rakyatnya
menderita karena banjir.
D. Sulit berdiskusi pada orang yang keras kepala.
E. Pemimpin bertangan besi itu kemarin meninggal dunia.

19. Ibu Ria adalah klien saya. Masih gesit, masih baca puisi bersama-sama teman seumurnya. Sekitar
dua minggu menjelang Natal saya bertemu dia di kantornya. Dia bercerita bahwa anaknya yang
akan membuat garage sale mengusulkan melego meja makan dan meja di ruang tamu rumahnya.

Yang dimaksud dengan garage sale adalah….

A. Obralan.
B. Pameran makanan.
C. Pesta di gudang.
D. Ulang tahun.
E. Reuni.

20. Ekspor timah dari Indonesia yang merupakan penghasil timah terbesar kedua di dunia setelah
Cina semakin berkurang setelah pemerintah menutup pertambangan ilegal di Bangka dan
Belitung. Upaya ini menjadikan timah sebagai salah satu komoditas yang berperforma terbaik di
London Metal Exchange pada tahun-tahun terakhir.

Kata upaya dalam teks ini sangat tepat bila diganti dengan ….

6
alternatifa

A. Usaha
B. Ikhtiar.
C. Kebijakan.
D. Kebijaksanaan
E. Tindakan.

7
alternatifa

UTUL UGM BAHASA INDONESIA 2010 461


1. Hingga saat ini, Amerika Serikat dan Cina masih menduduki peringkat teratas penyumbang emisi
dunia. Tingginya tingkat emisi kedua negara tersebut salah satunya diakibatkan oleh tingginya
penggunaan batu bara sebagai sumber energi bagi pembangkit listrik. Amerika Serikat masih
memiliki cadangan batu bara sebesar 264 miliar ton, dan cukup untuk memenuhi kebutuhan
energi 225 tahun ke depan dengan laju saat ini. Batu bara yang murah dan mudah diperoleh di
kawasan Amerika Serikat menghasilkan setengah daya listrik negara tersebut. Setiap hari sekitar
14.000 ton batu bara dibakar di pembangkit listrik Hunter, Utah yang memasok listrik ke wilayah
AS bagian barat. Guna memproduksi lebih dari 18 miliar ton karbon dioksida per tahun. Di Cina,
lebih dari 80% listrik negara dihasilkan oleh pembangkit bertenaga batu bara. Tercatat sebanyak
46% produksi batu bara tua dunia dihasilkan Cina. Bahkan, pada awal tahun 2008, dua buah
pembangkit listrik tenaga batu bara dioperasikan setiap minggunya untuk mengimbangi
pesatnya laju pertumbuhan ekonomi Cina.

Inti paragraf di atas adalah …

A. Hingga saat ini, Amerika Serikat dan Cina masih menduduki peringkat teratas negara
penyumbang emisi dunia.
B. Tingginya tingkat emisi di AS dan Cina salah satunya diakibatkan oleh tingginya penggunaan
batu bara sebagai sumber energi bagi pembangkit listrik.
C. Tercatat sebanyak 46% produksi batu bara tua dunia dihasilkan Cina.
D. Amerika Serikat dan Cina merupakan produsen terbesar emisi karbondioksida karena
tingginya penggunaan batu bara sebagai pembangkit tenaga listrik di negara tersebut.
E. Setiap hari sekitar 14.000 ton batu bara dibakar di pembangkit listrik Hunter, Utah, yang
memasok listrik ke wilayah AS bagian barat.

2. Pemerintah diharapkan segera membentuk panel khusus untuk mengaudit penambangan batu
bara di wilayah Kalimantan. Audit itu diperlukan untuk mengetahui sebaran penambangan,
kesesuaian lahan, dan kepatuhan pemilik kuasa pertambangan pada standar lingkungan. Sejauh
ini pemerintah daerah dan pusat dinilai saling tunggu untuk mengambil langkah yang harus
dilakukan guna mengatasi masalah penambangan yang merambah hutan konservasi di
Kalimantan. Pemerintah daerah menuding pemerintah pusat seenaknya mengeluarkan izin dan
menetapkan wilayah hutan, sementara pemerintah pusat menuduh pemda tidak terkendali
memberi kuasa pertambangan.

Yang tersebut di bawah ini dikemukakan dalam bacaan di atas, kecuali…

A. Penambangan batu bara meresahkan.


B. Penambangan mengabaikan standar lingkungan.
C. Sebaran penambangan tidak transparan.
D. Penambangan batu bara dibatasi.
E. Pemerintah pusat bebas mengeluarkan izin

3. Pemberian vaksin lewat goresan di kulit ternyata memberikan respons imun yang lebih kuat
dibandingkan dengan vaksin yang disuntikkan. Para peneliti juga menemukan bahwa vaksin
lewat goresan tubuh akan 100 kali lebih cepat membentuk zat kekebalan. Pemberian vaksin
dengan cara menggores kulit tersebut pertama kali dikenal pada dua abad lalu, yakni saat
pemberian vaksin pencegah cacar (smallpox). Namun, kini sebagian besar metode imunisasi

1
alternatifa

modern diberikan lewat injeksi. Dalam beberapa tahap penelitian, para ahli dari Brigham and
Women's Hospital, Amerika Serikat, juga menemukan bahwa cara pemberian vaksin yang
dilakukan dengan menggores tersebut akan membuat memori sel T, sel yang menjaga sistem
imun untuk melawan virus penyebab penyakit, meningkat. Sel T diduga lebih penting daripada
sel imun. Sel ini berada di daerah kelenjar getah bening, darah, dan jaringan periferal, seperti
kulit dan paru-paru.

Berdasarkan isi bacaan di atas, pernyataan berikut ini benar, kecuali…

A. Para peneliti juga menemukan bahwa vaksin lewat goresan tubuh akan 100 kali lebih cepat
membentuk kekebalan.
B. Brigham and Women's Hospital adalah tempat para periset meyakini temuannya bahwa sel
T ternyata lebih penting daripada sel imun.
C. Metode imunisasi modern yang kini lebih populer dan paling efektif dilakukan adalah
pemberian vaksin melalui cara injeksi.
D. Cara pemberian vaksin yang dilakukan dengan menggores tersebut akan membuat sistem
memori sel T, sel yang menjaga sistem imun untuk melawan virus penyebab penyakit,
meningkat.
E. Sel berada di daerah getah bening, darah, dan jaringan periferal, seperti kulit dan paru-paru.

4. Upaya meniadakan nyamuk penyebab demam berdarah dengue (DBD) hanya satu, yaitu
menyuluh masyarakat bukan air comberan atau sampah, melainkan air jernih tergenanglah yang
merupakan tempat nyamuk aedes bersarang. Oleh karena itu, segala yang berpotensi menjadi
sarang perindukan nyamuk wajib disingkirkan. Kebijakan mencegah bukan dengan menyemprot
(fogging), melainkan membunuh nyamuk dengan larvasida, mengingat umur nyamuk dewasa
tidak lebih panjang daripada jentiknya. Nyamuk dewasa baru disemprot kalau sudah ada yang
terjangkit

Inti paragraf di atas adalah …

A. Pemerintah harus memberi penyuluhan kepada masyarakat.


B. Penanganan wabah demam berdarah dengue (DBD) selama ini keliru.
C. Masyarakat harus diberi tahu cara mencegah DBD secara benar.
D. Hal yang penting dilakukan dalam penanganan DBD adalah menghilangkan jentik.
E. Fogging yang selama ini dilakukan tidak bisa berhasil mencegah DBD.

5. Seni, apa pun bentuknya, adalah media ekspresi yang secara estetik membangun watak dan
karakter manusia Indonesia dalam setiap wujudnya. Tak Jarang, identitas "manusia Indonesia"
ditemukan bukan dari mata pelajaran budi pekerti, kewarganegaraan, atau wawasan nusantara,
melainkan dari sebuah lakon yang dipentaskan Bengkel Teater, Teater Koma, atau Teater
Gandrik. "Kepribadian Indonesia" juga ditemukan pada novel-novel Andrea Hirata, Umar Kayam,
Mochtar Lubis, atau sajak-sajak Chairil Anwar, W.S. Rendra, Taufik Ismail, dan sederet nama lain
di negeri ini. Dengan demikian, sudah selayaknya negara berperan dalam menumbuhkan
kesadaran estetik guna menghargai produk-produk seni yang lahir di negeri ini.

Inti paragraf di atas adalah…

A. Indonesia mempunyai sejumlah nama besar di bidang kesenian yang layak dibanggakan.
B. Seni yang sudah terbukti membentuk kepribadian bangsa perlu mendapat perhatian
pemerintah.

2
alternatifa

C. Identitas Indonesia bisa ditemui pada pentas--pentas teater dan karya sastra.
D. Karya seni adalah pembentuk kepribadian yang lebih baik dibandingkan dengan mata
pelajaran di sekolah.
E. Mata pelajaran di sekolah tidak berhasil membentuk kepribadian bangsa Indonesia.

6. Masa lalu peradaban bukan hanya peristiwa dan pengalaman yang lewat. Masa lalu merupakan
bagian integral yang bertali-temali dengan masa sekarang dan masa depan. Masa lalu, sekarang,
dan masa nanti tidak bersifat linear. Dalam tiga entitas itu terletak harapan dan keputusasaan,
keberhasilan dan kegagalan. Belajar dari masa lalu tidak hanya bisa ditempuh lewat warisan
pemikiran dan tradisi tokoh bangsa, tetapi juga melalui warisan yang berupa benda mati, yang
ada di atas permukaan tanah atau yang ada di kolong tanah. Kita bangga dengan Muaro Jambi
atau Borobudur yang termasuk kategori warisan dunia, sekadar memungut dua contoh.

Inti paragraf di atas adalah…

A. Masa lalu merupakan sejarah dari masa kini.


B. Zaman selalu berkembang.
C. Kita harus belajar dari masa lalu jika ingin berhasil.
D. Masa lalu merupakan hal yang penting untuk dipelajari.
E. Benda mati di permukaan bumi merupakan warisan yang berharga.

7. Adat yang mengharuskan pria membayar dan melunasi mas kawin tanpa disadari menempatkan
perempuan Nabire, Papua, sebagai komoditas. Apalagi yang menentukan besarnya mas kawin
bukan perempuan, melainkan paman. Sejak awal posisi setara antara perempuan dan laki-laki
dalam perkawinan tidak terjadi, itulah yang menyebabkan perempuan Nabire rentan kekerasan
dalam lingkup domestik. Setelah berkeluarga, tugas perempuan kian berat. Selain melahirkan
dan mengurus anak, mereka juga harus mengelola kebun. Di pedalaman dengan mudah dijumpai
seorang ibu seraya membopong anak di pundaknya juga membawa tas rajut berisi ubi serta
memanggul sekop atau kapak.

Inti paragraf di atas adalah…

A. Perempuan di Papua mempunyai tugas mengurus anak dan mencari makan.


B. Dalam rumah tangga, kedudukan perempuan di Nabire di bawah laki-laki dan rentan
terhadap kekerasan.
C. Laki-laki di Nabire leluasa berbuat apa saja terhadap istrinya karena merasa sudah memberi
mas kawin.
D. Perempuan di Nabire mempunyai peran ganda sekaligus beban ganda.
E. Posisi setara antara laki-laki dan perempuan dalam masyarakat Nabire sulit ditemukan.

8. Kadar kolesterol tinggi banyak dialami oleh orang berbagai usia dan resikonya terus meningkat
seiring dengan bertambahnya usia. Kolesterol tinggi ini umumnya disebabkan oleh gaya hidup
tidak sehat, seperti makan makanan tidak sehat yang berlemak jenuh dan berkolesterol tinggi.
Merokok, kurang olahraga, dan stres pun memicu naiknya kadar kolesterol. Selain itu, faktor
genetik atau keturunan juga patut diwaspadai, seperti orang dengan hipertensi, obesitas,
diabetes melitus, serta mempunyai riwayat keluarga sekandung, karena potensial menjadi
pengidap hipertensi dan penyakit jantung koroner.

Paragraf di atas dapat diringkas menjadi …

3
alternatifa

A. Kolesterol adalah penyakit yang umum diderita.


B. Kolesterol meningkat seiring dengan bertambahnya usia.
C. Kolesterol disebabkan oleh gaya hidup tidak sehat.
D. Kolesterol merupakan penyakit turunan.
E. Kolesterol menyebabkan obesitas.

9. Bisnis menuntut inovasi. Inovasi tidak selalu merupakan penemuan baru atau teknologi baru,
tetapi dapat juga aplikasi baru dari hal-hal yang sudah dianggap kuno. Bisnis mengandung
kebutuhan yang terus-menerus untuk menjajaki sekeliling kita, menguji setiap ujung lemah yang
mungkin ditajamkan sebagai sebuah tombak bagi sebuah pusat laba baru. Sementara itu, sekolah
bisnis biasanya hanya mengajarkan hal-hal yang telah terjadi.

Ide pokok paragraf di atas adalah …

A. Teknologi baru dalam bisnis.


B. Perlunya sekolah dalam pengembangan bisnis.
C. Penerapan hal yang baru dari sesuatu yang sudah ada.
D. Perlunya inovasi untuk meraih laba dalam bisnis.
E. Perlunya pembaruan dalam bisnis.

10. Anda ingin berwisata alam pedesaan sambil menikmati hidangan istimewa, memandang
hamparan sawah luas terbentang dan deretan pohon kelapa yang membingkai tepian desa,
mendengar suara gemericik air dan menyaksikan ikan-ikan yang, berenang bebas di kolam alam,
sambil menikmati masakan udang galah yang lezat.

Inti kalimat di atas adalah …

A. Anda ingin pergi ke desa?


B. Anda ingin menikmati pemandangan alam?
C. Anda ingin menikmati udang galah yang lezat?
D. Anda ingin berwisata?
E. Anda ingin berwisata alam dan berwisata kuliner?

11. Pemberian nama diri terhadap seseorang rupanya tidak sesederhana yang disangka orang. Pada
umumnya, penamaan disesuaikan dengan pertimbangan-pertimbangan tertentu, seperti menjadi
penanda identitas, sebagai sarana penghormatan, penanda urutan, penanda jenis kelamin,
penanda keakraban, kerahasiaan, gengsi, dan sebagainya. Dalam hal ini, ungkapan Shakespeare
yang berbunyi "apalah artinya sebuah nama tampaknya tidak berlaku bagi, masyarakat dan perlu
direnungkan kembali kebenarannya.”

Paragraf ini membahas …

A. Pemberian nama diri.


B. Pertimbangan penamaan diri.
C. Nama diri sebagai identitas.
D. Pandangan Shakespeare
E. Kerumitan penamaan diri.

12. Hingga kini, kurangnya tenaga medis dan kesenjangan fasilitas kesehatan, terutama di pedesaan,
masih terjadi. (…) pemerintah perlu memberikan dorongan pada para calon dokter dan

4
alternatifa

paramedis agar bersedia ditempatkan di berbagai puskesmas atau rumah sakit di daerah
terpencil (…), pemerintah perlu memberikan perhatian dan penghargaan yang layak pada tenaga
medis yang mengabdi di daerah tertinggal, (…) diharapkan, (...) masyarakat desa (...) kota sama-
sama menikmati pelayanan kesehatan.

Kata yang tepat untuk melengkapi titik-titik dalam paragraf di atas berturut-turut adalah …

A. karena, sehingga, adapun, baik, ataupun.


B. oleh karena itu, untuk itu, dengan demikian, baik, maupun.
C. di samping ilu, sehingga, oleh sebab itu, baik.
D. selain itu, sehingga, maka, baik, ataupun.
E. sedangkan, sehingga, adapun, baik, atau.

13. Walikota Yogyakarta sudah memberikan berbagai kelonggaran kepada para pedagang kaki lima
untuk berjualan di trotoar Jalan Malioboro atau tempat-tempat lain selama bulan puasa.

Kalimat yang sama strukturnya dengan kalimat di atas adalah…

A. Gubernur DKI menertibkan para gelandangan dan pengemis yang berumah di bawah
jembatan.
B. Rektor UI memberikan beasiswa kepada mahasiswa berprestasi dari berbagai fakultas.
C. Presiden memberi arahan kepada para menteri untuk tetap bekerja keras tanpa terganggu
isu-isu pemakzulan.
D. Para mahasiswa menjadi pendamping anak-anak jalanan di lembah Kali Code.
E. Menteri Sosial RI menggelontorkan dana usaha kepada pemuda-pemuda lulusan SMK untuk
berwiraswasta dalam berbagai bidang sesuai dengan minatnya.

14. Kalimat yang benar adalah …


A. Untuk mengekplorasi dan mengekploitasi minyak dan gas bumi yang mana merupakan
sumber devisa negara diperlukan tenaga ahli di bidang geologi dan perminyakan.
B. Untuk mengekplorasi dan mengekploitasi minyak dan gas bumi, mengingat sebagai sumber
devisa negara memerlukan tenaga ahli yang terampil di bidang geologi dan perminyakan.
C. Untuk mengekplorasi dan mengekploitasi minyak dan gas bumi karena sebagai sumber
devisa negara diperlukan tenaga ahli yang terampil di bidang geologi dan perminyakan.
D. Untuk mengekplorasi dan mengekploitasi minyak dan gas bumi sebagai sumber devisa
negara memerlukan tenaga ahli yang terampil di bidang geologi dan perminyakan.
E. Untuk mengeksplorasi dan mengeksploitasi minyak dan gas bumi yang merupakan sumber
devisa negara diperlukan tenaga ahli yang terampil di bidang geologi dan perminyakan.

15. Karena terbukti menguntungkan, sekarang banyak perusahaan asing berbondong-bondong


menanamkan modalnya di perkebunan kelapa sawit, seperti RCMA dari Inggris, Uni Royal dari
AS, dan SIPEF dari Belgia.

Inti kalimat di atas adalah …

A. Berkebun kelapa sawit menguntungkan.


B. Perusahaan asing menanamkan modalnya.
C. Perusahaan asing mendapatkan keuntungan.
D. Kelapa sawit laku keras di Inggris, AS, dan Belgia

5
alternatifa

E. Perkebunan kelapa sawit menarik modal asing.

16. Tertawa dapat menghambat aliran kortisol, hormon stres yang dapat menyebabkan peningkatan
tekanan darah, penggumpalan darah, dan penurunan sistem kekebalan tubuh.

Inti kalimat di atas adalah…

A. Tertawa menurunkan tekanan darah.


B. Tertawa menyebabkan penggumpalan darah.
C. Tertawa meningkatkan hormon stres.
D. Tertawa meningkatkan sistem kekebalan tubuh.
E. Tertawa menghambat aliran kortisol.

17. Untuk menjaga keberlangsungan sektor pertanian, pemerintah perlu menciptakan suatu keadaan
yang mendorong kaum muda tertarik bekerja di sektor ini.

Pola kalimat di atas sama dengan pola kalimat…

A. Dalam memakai alat-alat teknologi komunikasi yang canggih, kita harus memperhatikan
akibat yang berpengaruh negatif.
B. Kenaikan harga jual produk pertanian dapat mendorong anak-anak muda terjun ke sektor
ini.
C. Kurangnya minat kaum muda bergelut dengan cangkul dan tanah tampaknya sudah
menggejala di Pulau Jawa.
D. Dalam usahanya memenangkan perkara, kadang kadang para pengacara tidak lagi
mempertimbangkan kebenaran, tetapi mementingkan siapa yang membayar.
E. Sepanjang dua belas ronde Tyson terus- menerus memukuli lawannya, tetapi tidak juga bisa
meng-KO.

18. Makna asosiasi dapat ditemukan dalam kalimat-kalimat berikut, kecuali …


A. Mereka sering mengolok-oloknya karena berasal dari Ragunan.
B. Beberapa saksi mata mengatakan bahwa pada hari-hari tertentu di sekitar pohon itu sering
terlihat putih-putih melintas dengan cepat.
C. Mulut gang itu telah ditutup pagar kawat berduri agar para perusuh tidak dapat masuk ke
perkampungan.
D. Sejak 1976 warung Pak Kumis memang telah digandrungi orang.
E. Dengan khidmat semua peserta upacara memberi hormat kepada Sang Merah Putih.

19. Pemakaian kalau secara tepat dapat dijumpai dalam kalimat…


A. Rencana pemberian insentif untuk anggota dewan selama sidang Pansus Century dapat
ditinjau ulang kalau dipandang perlu.
B. Banyak SMK di Yogyakarta tidak menyangka kalau anggaran uji kompetensi tidak turun
tahun ini sehingga pencairan dana untuk melaksanakannya sebagian besar dibebankan
kepada siswa.
C. Banyak orang tidak menyangka kalau bank yang relatif tidak terlalu besar itu diselamatkan
dengan bail-out yang jumlahnya tidak rasional.
D. Sering tidak disadari kalau uploading foto-foto seronok para remaja di Facebook dapat
mengundang prasangka negatif.

6
alternatifa

E. Para pembalak tidak mau tahu kalau aktivitas yang dilakukannya itu bisa berakibat fatal
terhadap ekosistem beberapa tahun kemudian.

20. Penulisan kalimat berikut tidak sesuai dengan kaidah EYD, kecuali…
A. Dunia kita sekarang memang jauh lebih komplek dibandingkan dengan masa yang lalu
karena, tehnologi yang makin canggih telah membantu orang lebih efisien dan cerdas, juga
lebih terbuka.
B. Kini di era perkembangan teknologi informasi, kita dengan mudah bisa diteropong dan
meneropong pendapat, pemikiran, emosi, kedalaman etika, bahkan tatakrama kita lewat
status di facebook, YM, juga forum diskusi di berbagai situs di internet.
C. Gus dur telah berpulang, kata rohaniawan itu, "Tetapi ajarannya tentang demokrasi dan
toleransi beragama masih perlu dikembangkan dalam masyarakat kita yang multi kultural.
D. Meskipun sering kontroversial, pengaruh Gus Dur terbukti masih sangat kuat dan diikuti
oleh ribuan santrinya.
E. Presiden SBY mengatakan, dalam penyusunan draft teks baru yang mengakomodasi
masukan para kepala negara pada pertemuan Konferensi Perubahan lklim PBB harus
dicantumkan keseimbangan kemitraan yang adil antara negara maju dan berkembang
dalam pengelolaan hutan.

7
alternatifa

UTUL UGM BAHASA INDONESIA 2013 25


1. Penulisan huruf kapital yang mengikuti Perindustrian MS Hidayat saat ini
EYD terdapat dalam kalimat berikut …. dinilai hanya masih sebatas
A. Senin malam banjir lahar kembali inventarisasi permasalahan dunia
terjadi di aliran Kali Putih dan usaha.
sempat menerobos areal dekat D. Pada Selasa dilaporkan bahwa
pasar Jumoyo. terjadinya letusan sekunder di Kali
B. Setelah sejak lama diserahkan ke Gendol yang menyebabkan
DPR RI pemerintah melalui munculnya asap dengan ketinggian
Mendagri akan memberikan sekitar 300 meter.
penjelasan mengenai rancangan E. Pada bulan Desember juga akan ada
Undang-Undang Keistimewaan DIY hari besar keagamaan dan Tahun
di hadapan Komisi II. Baru sehingga penggunaan
C. Enam partai kecil di Salatiga akan premium akan naik.
menggetarkan Pemilihan Umum
Kepala Daerah (Pemilukada) kota 3. Kalimat berikut adalah kalimat baku…..
Salatiga yang digelar pada 8 Mei A. Partai politik yang tidak mentaati
2013 mendatang. peraturan akan mendapatkan
D. Sebagaimana diungkapkan oleh sanksi tegas.
Jassin surat dari PSSI yang berisi B. UGM memelopori aktivitas-
penegasan bahwa Piala AFF aktivitas itu dengan mengadakan
bukanlah gratifikasi itu telah KKN pengamat pemilu.
diterima KPK. C. Dealer menyediakan beberapa
E. Dalam usaha menegakkan aturan kemudahan dalam pembayaran
dan menghilangkan kesan tidak adil sehingga konsumen dapat menyicil
Komisi Penyiaran Indonesia Daerah angsuran sesuai waktu yang
Jateng akan menertibkan Radio ditentukan.
Siaran Pemerintah Daerah. D. Meski tidak diakui, beberapa tahun
lalu Golkar pernah
2. Penulisan huruf kapital yang benar mengintruksikan kantor-kantor
terdapat dalam kalimat … pemerintah untuk mencat pagar
A. Aspek infrastruktur pula yang mereka dengan warna kuning.
membuat penerapan pembatasan E. Dengan adanya status otonomi
premium tidak bisa serentak pemerintah pusat tidak lagi
dilakukan di seluruh Tanah Air. sepenuhnya mensubsidi daerah.
B. Balai Penyelidikan dan
Pengembangan Teknologi 4. Mengharap kehadiran Bapak/Ibu pada
Kegunungapian Yogyakarta acara syukuran pernikahan anak kami
memperkirakan bahwa bahaya yang akan kami selenggarakan pada
lahar dingin dari material vulkanik Rabu, 2 Februari 2011.
hasil erupsi Gunung Merapi akan
Kalimat di atas tidak memenuhi syarat
berlangsung dalam waktu yang
sebagai struktur kalimat yang baku
lama.
karena ...
C. Sosialisasi peningkatan
penggunaan produk dalam negeri A. Penggunaan kata kami yang
(P3DN) yang dilakukan Menteri berlebihan.

1
alternatifa

B. Tidak berobjek. D. Arisan tidak hanya dilakukan oleh


C. Tidak bersubjek. ibu-ibu dikampung, melainkan juga
D. Tidak bersubjek dan tidak berobjek. para pemulung, pengasong, dan
E. Tidak menggunakan bentuk aktif kondektur bus kota.
pada kata selenggarakan. E. Ada sinyalemen menyatakan
peredaran narkoba cenderung
5. Berikut ini merupakan kalimat majemuk meningkat seiring dengan
bertingkat …. maraknya bisnis hiburan malam.
A. Dari sudut pandang ini, kegiatan
pembelajaran tidak lagi bersifat 7. Induk kalimat yang berintikan
klasikal, tetapi lebih dikembangkan pronomina dan nomina terdapat pada ….
ke arah kerja kelompok dan A. Sebagai tanda kasih dan terima
perseorangan. kasih, merekalah yang melayani
B. Anak-anak perlu mempersiapkan para ibu.
diri untuk memasuki era B. Karang yang mengalami pemutihan
demokratisasi, suatu era yang total umumnya dari spesies
ditandai oleh keragaman perilaku, seriatopora, sedangkan yang
dengan cara terlibat dan mengalami pemutihannya moderat dari jenis
secara langsung proses pocillopora.
pendemokrasian. C. Dari skenario itu bisa
C. Dalam pendaftaran sekolah pun dikembangkan identifikasi sumber
calon siswa akan dirangking dan daya yang tersedia.
dikelompokkan berdasarkan nilai D. Karang yang lambat memulihkan
ujian nasional. diri dengan mudah digantikan oleh
D. Sekolah yang seharusnya menjadi karang yang pemulihannya lebih
tempat yang menyenangkan dan cepat.
membudayakan anak berubah E. selalu saja ada pihak-pihak yang
menjadi arena persaingan yang mengambil sikap ekstrem.
tidak sehat.
E. Misi pencerahan pendidikan harus 8. Makna kolusi dapat ditemukan dalam
mencakup perbaikan aspek-aspek kalimat berikut….
kemiskinan tersebut secara A. Perusahaannya berkembang pesat
seimbang dan tidak dapat direduksi karena ia pandai menjaga
pada orientasi kemajuan ekonomi hubungannya dengan banyak rekan
semata. bisnisnya.
B. Budaya “pertemanan” dituding
6. Kalimat berikut adalah kalimat baku…. banyak orang telah menyuburkan
A. Cukup banyak partai politik yang aktivitas kerja sama tak resmi
masih mengkultuskan Bung Besar dalam melancarkan urusan di
sebagai maskot perjuangan mereka. berbagai bidang birokrasi.
B. Selain mengorganisasikan materi, C. Banyak asumsi menyatakan bahwa
guru harus dapat memposisikan kekayaan yang dipermasalahkan
dirinya sebagai aktor yang baik itu didapatnya dengan cara yang
agar kelas menjadi hidup. tidak dapat
C. Dibandingkan dengan GM, T dipertanggungjawabkan dari
menjual lebih banyak mobil di luar banyak pos kegiatan pemerintahan.
Amerika Serikat.

2
alternatifa

D. Juru kampanye partai harus pandai anugerah dari Tuhan kepada


berdiplomasi untuk menarik massa makhluknya, hak asasi tidak dapat
sebanyak-banyaknya agar dapat dipisahkan dari eksistensi pribadi
memenangkan pemilu. manusia itu sendiri. (3) Hak asasi tidak
E. Pendapat-pendapat bernada minor dapat dicabut oleh suatu kekuasaan atau
bermunculan manakala seorang oleh sebab-sebab lainnya, (4) Karena jika
presiden mengangkat putrinya hal itu terjadi maka manusia kehilangan
dalam jajaran menteri kabinetnya. martabat yang sebenarnya menjadi inti
nilai kemanusiaan. (5) Walau demikian,
9. Penggunaan kata terukir yang bermakna bukan berarti bahwa perwujudan hak
leksikal terdapat dalam kalimat .... asasi manusia dapat dilaksanakan secara
A. Kenangan indah semasa di SMA mutlak karena dapat. (6) Melanggar hak
terukir dihatinya. asasi orang lain. (7) Memperjuangkan
B. Masih terukir dengan sangat jelas hak sendiri sampai-sampai mengabaikan
diingatannya peristiwa letusan hak orang lain, ini merupakan tindakan
Gunung Merapi itu. yang tidak manusiawi. (8) Kita wajib
C. Suasana hiruk pikuk di tempat menyadari bahwa hak-hak asasi kita
pengungsian terukir dengan baik selalu berbatasan dengan hak-hak asasi
pada lukisannya. orang lain.
D. Masa remaja yang penuh cerita
Di dalam paragraf tersebut terdapat
terukir begitu mendalam di hatinya.
kalimat yang dapat dihapuskan tanpa
E. Jejak-jejak ombak serta alur air
mengurangi estimasinya, yaitu kalimat
terlihat terukir di atas karang.
….
10. Masyarakat masa depan yang berada A. ke-1
dalam konteks risk society perlu B. ke-2
melakukan kalkukasi risiko. Kalkulasi ini C. ke-3
akan membantu untuk mengarahkan D. ke-4
masyarakat ke format masyarakat masa E. ke-5
depan yang diinginkan melalui prediksi
yang disusun dan keputusan yang 12. Harga karet alam di pasar dunia tahun
diambil. 2013 Tetap bagus. Seiring meningkatnya
produksi kendaraan, permintaan
Makna risk society dalam paragraf di atas
terhadap karet meningkat. Menurut
adalah….
Chairman Gabungan Perusahaan Karet
A. Masyarakat yang berisiko. Indonesia Asril Sutan Amir, tingginya
B. Risiko bermasyarakat. permintaan mendorong peningkatan
C. Risiko kemasyarakatan. laju ekspor karet. Ekspor karet tahun
D. Masyarakat berisiko. 2013 diperkirakan meningkat 6 – 8
E. Risiko yang harus ditanggung persen dari total ekspor tahun 2012,
masyarakat. atau mencapai 2,4 juta ton. Ekspor karet
tahun 2012 hingga Oktober adalah
11. (1) Hak asasi manusia dalam pengertian sebanyak 2, 01 juta ton. Hingga akhir
umum adalah hak-hak dasar yang 2012 ekspor karet bertambah 400.000
dimiliki oleh setiap pribadi manusia ton.
sebagai anugerah Tuhan yang dibawa
sejak lahir. (2) Ini berarti bahwa sebagai

3
alternatifa

Pernyataan berikut ini sesuai dengan isi A. Sarana tidak memadai


yang terkandung dalam paragraf di atas. B. Harga barang mahal
A. Pada tahun 2013 diperkirakan C. Arus lalu lintas seret
harga karet alam akan naik. D. Listrik byar-pet
B. Pada tahun 2013 terjadi E. Jalanan rusak
peningkatan permintaan terhadap
karet. 15. Akibat erupsi Merapi, seluruh anggota
C. Ekspor karet pada tahun 2013 keluarganya tidur berhari-hari di
meningkat di atas 8 persen. pegunungan.
D. Permintaan ekspor karet terutama
dipengaruhi oleh meningkatnya Bentuk ulang yang bermakna sama
produksi kendaraan. dengan bentuk ulang yang terdapat pada
E. Ekspor karet sampai akhir tahun kalimat di atas adalah …
2010 mencapai 2,5 juta ton. A. Meskipun telah dibacanya
berhalaman-halaman, dia belum
13. Daging termasuk jenis makanan yang menemukan juga apa yang
punya segi negatif dan positif. Daging dicarinya.
mengandung protein dan zat besi yang B. Kakek terkekeh-kekeh mendengar
penting untuk tubuh, tetapi lemaknya ceritaku.
yang menjadikan daging sebagai musuh C. Ketika kami beristirahat di lembah
nomor satu bagi kesehatan. Karena itu, itu terdengar suara burung
asupan daging dalam menu harian perlu bersahut-sahutan.
dibatasi, maksimal sekali seminggu. D. Setelah bersalam-salaman kami
makan bersama sambil
Isi paragraf di atas adalah ….
melanjutkan obrolan.
A. Segi positif dan negatif daging. E. Sebelum mengajar, mereka terlebih
B. Kandungan protein dan zat besi dahulu mengikuti diklat selama dua
pada daging. bulan berturut-turut.
C. Kandungan lemak dalam daging
cukup tinggi. 16. Belakangan ini mafia hukum dan mafia
D. Asupan daging cukup dikonsumsi pajak sering dibahas, juga disorot mafia
seminggu sekali. pengadilan, pertambangan, dan
E. Pembatasan menu harian daging. narkotika. Keadaannya terasa absurd
karena perbincangan tentang bahaya
14. Jalanan rusak, berlubang di sana-sini, mafia hanya menimbulkan kegaduhan
dan tambahan aspal yang membuat lalu dan kehebohan ketimbang menggalang
lintas tak nyaman sudah menjadi upaya keras untuk mengatasinya. Semua
pemandangan biasa di negeri ini. Kondisi seperti tampak tidak berdaya. Tidak
ini jelas tak menguntungkan bagi hanya masyarakat yang mengeluh, tetapi
masyarakat dan dunia usaha. Harga juga pejabat pemerintah. Apakah negara
barang menjadi tambah mahal karena kalah menghadapi mafia?
arus lalu lintas komoditas yang seret.
Terdapat kesalahan pada kalimat-
Tak Cuma jalan, kualitas infrastruktur
kalimat pembentuk paragraf di atas,
lain juga sama saja seperti listrik masih
yaitu pada ....
byar-pet.

Inti paragraf itu adalah ….

4
alternatifa

A. Tiadanya tanda koma (,) setelah pemulihan kualitas lingkungan, serta


kata ‘pertambangan’ pada kalimat ekosistem pesisir. Peran penyuluh
kedua. perikanan selayaknya dihidupkan lagi
B. Mestinya tidak ada partikel ‘-nya’ sebagai garda terdepan arus informasi
pada kata ‘keadaannya’ pada kepada nelayan dan pembudidaya ikan.
kalimat kedua. Penyuluh juga memiliki peran strategis
C. Kata ‘tampak’ pada kalimat untuk melaporkan perkembangan
keempat mestinya diganti dengan lapangan sehingga kebijakan yang
kata ‘nampak’. diambil pemerintah bisa menyentuh
D. Kata penghubung ‘tetapi’ pada sasaran.
kalimat kelima mestinya diganti
Pokok persoalan yang dibahas dalam
dengan ‘melainkan’.
paragraf diatas adalah …
E. Kata ‘Apakah’ pada kalimat keenam
mestinya diganti dengan ‘Mengapa’. A. Masalah kerentanan sektor
perikanan.
17. Dalam masyarakat Jawa, wayang B. Masalah keseriusan pemerintah
merupakan seni pertunjukan yang dalam pembangunan perikanan.
mempunyai peran sebagai sarana C. Masalah pembangunan perikanan
edukatif dan refleksi filosofis. Wayang yang kurang mengacu pada
yang melakonkan cerita berasal dari kenyataan yang terjadi di lapangan.
India, yaitu dari epos Ramayana dan D. Masalah peran penyuluh perikanan.
Mahabarata, itu menjadi seni lokal yang E. Masalah keseriusan pemerintah
mencerminkan budaya asli karena dalam mengatasi problem yang
perubahan-perubahan yang dilakukan dihadapi nelayan dan
terhadap cerita sumbernya. Hazeau, pembudidayaan ikan.
seorang pakar budaya Jawa, mengatakan
bahwa “The Javanese theatre and the 19. Sudah cukup lama upaya pemberantasan
Javanese plays are technically original mafia hukum diwacanakan oleh para
Javanese, not revealing any Indian penegak hukum. Ironisnya, meskipun
influence”. wacana tersebut bergaung cukup kuat,
pihak aparat jarang bisa menangkap
Paragraf itu dapat diringkas sebagai
mafiosonya. Hal itu karena sosok
berikut…
mafiosonya tidak pernah jelas. Dalam
A. Wayang adalah kesenian asli Jawa pengalaman banyak negara, karena
B. Wayang menjadi seni lokal mafia memang merupakan organisasi
C. Wayang berasal dari India kejahatan yang tertata rapi, mafiosonya
D. Wayang bercerita tentang epos sulit dilacak. Mafioso justru sering tampil
Ramayana atau menampilkan diri sebagai orang
E. Wayang selalu berubah terhormat. Bahkan, kata mafioso sendiri
sering diartikan sebagai “manusia
18. Dalam kondisi kerentanan sektor terhormat”.
perikanan, porsi pembangunan
Kalimat yang paling mencerminkan isi
perikanan selayaknya didekatkan pada
paragraf diatas adalah …
realitas keseharian nelayan. Keseriusan
pemerintah memperkuat nelayan harus A. Meskipun upaya pemberantasan
bisa dibuktikan dengan konsistensi mafia hukum telah diwacanakan,
penguatan nelayan, pengawasan dan karena mafiosonya tidak pernah

5
alternatifa

jelas, justru sering tampil sebagai


orang terhormat, upaya itu gagal
dilakukan.
B. Karena sosok mafiosonya tidak
pernah jelas, upaya pemberantasan
mafia hukum sering bersifat ironis
saja apalagi mafioso sering
menampilkan diri sebagai orang
terhormat.
C. Sebagai organisasi kejahatan yang
tertata rapi, mafia hukum tidak
pernah bisa diberantas.
D. Pihak aparat jarang bisa
menangkap mafioso karena
sosoknya tidak jelas.
E. Mafioso sering menampilkan diri
sebagai orang terhormat.

20. Di era globalisasi yang penuh persaingan


ini setiap orang dituntut untuk hidup
lebih dinamis, lebih aktif, dengan
mobilitas yang tinggi. Pola hidup
masyarakat yang sibuk ini pun semakin
berisiko mendatangkan stres dan
kelelahan fisik maupun psikis yang
menyebabkan gampang terserang
penyakit. Di sisi lain, kondisi lingkungan
semakin buruk dengan meningkatnya
pencemaran air dan udara.

Ide pokok paragraf itu adalah …

A. Globalisasi
B. Pola hidup
C. Persaingan
D. Kelelahan
E. Pencemaran

6
alternatifa

UTUL UGM BAHASA INDONESIA 2014


Bacaan berikut digunakan untuk menjawab soal nomor 1 sampai dengan nomor 3.

Etnis Sasak, Lombok, Nusa Tenggara Barat, memiliki kekayaan seni budaya yang beragam.
Salah satu di antaranya adalah naskah lontar atau takepan yang masih banyak disimpan masyarakat.
Naskah lontar antara lain berisi ajaran akhlak, cerita, falsafah, upacara adat, keagamaan, dan solidaritas
sosial. Naskah ini beraksara Jawa Kuno, Jawa, Bali, dan Sasak atau Jejawan. Keberadaan naskah ini
kemudian melahirkan tradisi membaca naskah lontar.

Tradisi membaca naskah lontar biasanya diselenggarakan pada acara tertentu, seperti
khitanan, ngurisang atau potong rambut pada anak balita, dan pernikahan. Naskah yang dibacakan
pada beragam acara tersebut disesuaikan dengan permintaan si empunya hajat. Dalam acara
ngurisang, naskah yang dibacakan bisa takepan indarjaya, sedangkan dalam acara pernikahan dapat
dibacakan naskah Dewi Rengganis. Takepan lndarjaya berisi kisah putri bisu yang kemudian bisa
berbicara setelah disapa seorang pemuda. Adapun naskah Dewi Rengganis berisi pedoman muda-mudi
dalam memilih jodoh.

Salah satu pembaca takepan yang relatif populer adalah Nari alias Amak Nurmini atau Mini.
Usianya sudah lanjut, 70 tahun. Namun, ia masih sering kali mendapat permintaan warga Lombok untuk
membaca takepan demi meramaikan acara yang mereka selenggarakan.

1. Ide pokok bacaan di atas adalah …


A. Kekayaan budaya Lombok.
B. Membaca naskah lontar sebagai tradisi di Lombok.
C. Pembaca lontar yang populer di Lombok.
D. Isi naskah lontar Lombok.
E. Perlunya pelestarian naskah lontar Lombok.

2. Inti bacaan di atas adalah …


A. Membaca naskah lontar menjadi tradisi di Lombok.
B. Tradisi membaca naskah lontar sudah mulai ditinggalkan.
C. Naskah lontar memiliki banyak ajaran.
D. Membaca naskah lontar dilakukan dalam acara tertentu.
E. Pembaca naskah lontar sudah Ianjut usia.

3. Pernyataan yang berhubungan dengan isi bacaan di atas adalah …


A. Naskah lontar tergerus oleh kemajuan teknologi.
B. Ada banyak naskah lontar yang hilang.
C. Generasi muda sudah tidak tertarik membaca naskah lontar.
D. Nari adalah salah seorang pembaca naskah lontar.
E. Ada lomba membaca naskah lontar.

Badan Kesehatan Dunia (WHO) menyatakan bahwa polusi udara menjadi ancaman kesehatan terbesar
di dunia karena telah menyebabkan tujuh juta kematian pada tahun 2012. Polusi udara telah menjadi
satu dari delapan penyebab kematian di seluruh dunia pada tahun 2012. WHO menegaskan bahwa
upaya pengurangan polusi di dalam dan di luar rumah dapat menyelamatkan jutaan nyawa di masa

1
alternatifa

depan. Kematian akibat polusi udara biasanya terjadi dalam bentuk penyakit jantung, stroke, penyakit
paru-paru kronis, infeksi saluran udara , dan kanker paru-paru.

4. Inti paragraf di atas adalah …


A. Karena polusi udara telah menyebabkan tujuh juta kematian pada 2012, upaya pengurangan
polusi udara menjadi sangat penting.
B. Polusi udara merupakan salah satu penyebab kematian di seluruh dunia pada tahun 2012.
C. Menurut WHO, upaya pengurangan polusi di dalam dan di luar rumah dapat menyelamatkan
jutaan nyawa di masa depan.
D. Menurut WHO, pada tahun 2012 kematian yang disebabkan oleh polusi udara naik dua kali
lipat dari perhitungan tahun sebelumnya.
E. Menurut WHO, polusi udara menjadi ancaman kesehatan terbesar di dunia.

Orang-orang yang mempunyai hobi memancing ikan sudah tidak perlu lagi menunggu lama agar sang
ikan memakan umpan. Saat ini sudah ditemukan alat pencari ikan dengan teknologi sonar yang
dinamakan Fish Hunter Sonar. Alat ini dapat melacak kedalaman, temperatur, dan lokasi tempat
terdapat banyak ikan. Untuk mendapatkannya, pertama yang harus dilakukan adalah mengunduh
aplikasi untuk iOS. Setelah itu, pasang aplikasi yang telah diunduh tersebut. Selanjutnya, hubungkan
sonar dengan koneksi bluetooth ke smartphone. Alat ini siap dipakai dengan cara melepaskannya ke
target yang diincar (laut, danau, atau tempat lain yang diprediksi terdapat banyak ikan). Setelah
dilepaskan, perangkat ini dapat langsung mengirimkan informasi melalui smartphone mengenai
keberadaan ikan.

5. Ide pokok paragraf di atas adalah …


A. Ditemukannya alat pencari ikan otomatis.
B. Alat pencari ikan dengan teknologi sonar.
C. Pelacakan keberadaan ikan.
D. Prosedur pemakaian alat pencari ikan.
E. Alat pencari ikan otomatis siap pakai.

Efisiensi dapat dicapai oleh PT Pelni dengan menekan biaya operasional secara signifikan. Melalui audit
teknologi dengan memasang alat pengukur kinerja mesin penggerak dan generator di kapal akan bisa
diketahui kebocoran, keausan, dan kerusakan lainnya. Efisiensi juga bisa dilakukan dengan pengelolaan
kapal yang ramah lingkungan, antara lain dengan menggunakan bahan bakar nabati dan mendaur ulang
air limbah cair di kapal. Audit energi oleh BPPT pada gedung dan pabrik merekomendasikan perbaikan
sistem operasi yang dapat menghemat hingga 30% dari total konsumsi energi.

6. Pernyataan berikut ini yang tidak berhubungan dengan isi paragraf di atas adalah …
A. Pengelolaan kapal yang ramah lingkungan dapat menekan biaya operasional.
B. Alat pengukur kinerja mesin penggerak dan generator di kapal dapat mendeteksi
kebocoran, keausan, dan kerusakan lain.
C. Perbaikan sistem operasi dapat menghemat hingga 30% total konsumsi energi.
D. Audit energi dilakukan oleh BPPT pada gedung dan pabrik.
E. Bahan bakar nabati dan daur ulang air limbah cair di kapal dilakukan oleh PT Pelni.

2
alternatifa

Banjir bukan ancaman linier. Sebab musababnya berkembang mengikuti perilaku manusia. Menurut
catatan Badan Nasional Penanggulangan Bencana, pada era sebelum 1970-an penyebab banjir di DKI
didominasi oleh faktor alam. Pada era sesudahnya, faktor manusialah yang paling dominan. Perbuatan
manusia itu dari merusak daerah resapan air dan daerah aliran sungai sampai membuat permukiman
yang mengakibatkan penyempitan batang sungai.

7. Simpulan paragraf di atas adalah …


A. Karena itu, penyebab banjir berbeda-beda.
B. Dengan demikian, manusialah yang menjadi penyebab terjadinya banjir.
C. Oleh karena itu, banjir dapat ditanggulangi.
D. Jadi, ada perubahan cara menangani penyebab banjir.
E. Sehingga, penanggulangannya harus sesuai penyebabnya.

8. Pemakaian huruf kapital yang tepat terdapat dalam kalimat …


A. Karena sedang ada perbaikan, Saudara dimohon memakluminya.
B. Moh. Hatta adalah Wakil Presiden RI Pertama.
C. Daerah ini merupakan penghasil gula merah atau gula Jawa.
D. Kementerian Pendidikan Dan Kebudayaan membuka posko pengaduan ujian nasional.
E. Untuk menjaga stabilitas di kawasan Asia tenggara, dibentuklah organisasi yang bernama
ASEAN.

9. Penulisan unsur serapan yang benar terdapat pada kalimat …


A. Perbankan syariah hingga kini masih mengalami masalah lekuiditas.
B. Berdasarkan data Autoritas Jasa Keuangan, hingga akhir Desember 2013, angka Finance to
Deposit Ration (FDR) Bank Syariah mencapai 121 persen.
C. Yang tak kalah penting adalah penegakan substansi HAM itu sendiri, yaitu implementasi
konkrit dalam penyelesaian kasus-kasus pelanggaran berat HAM di masa lalu.
D. Direktur Utama BNI Syariah yakin bahwa kondisi perbankan syariah masih kondusif meski
pertumbuhannya tak setinggi 2013.
E. Dalam mengajukan budaya yang akan dinominasikan kita perlu menerapkan strateji seperti
membandingkan dengan apa yang diajukan negara lain.

10. Pemakaian tanda koma yang tidak tepat terdapat dalam kalimat…
A. Usaha diet dan olahraga yang dilakukannya dapat menurunkan berat badan, tetapi setelah
itu angka timbangan seolah sulit bergerak turun lagi.
B. Sebagai orang yang sibuk, pola makan yang tak seimbang sepertinya sudah menjadi
kebiasaan.
C. Makan dapat menimbulkan perasaan nyaman, sehingga banyak orang memilih makan saat
mereka sedang stress atau tertekan.
D. Profesor David Herman, peneliti di Universitas Gadjah Mada, melakukan riset itu selama
puluhan tahun
E. Karya seni pada dasarnya, merupakan hasil penafsiran kehidupan yang dilakukan oleh para
seniman melalui proses kreatif.

11. Pemakaian kata ulang yang tepat terdapat dalam kalimat …


A. Dengan beberapa kali membaca-baca tulisan di papan iklan, secara langsung anak belajar
membaca dan memahami isi bacaan.
B. Ada banyak surat-surat kabar terbit sehubungan dengan kebebasan pers.

3
alternatifa

C. Mereka berdua saling panggil-memanggil di bandara


D. Sayur-mayur dijual di pasar tradisional yang letaknya tidak jauh dari sini.
E. Angin puting beliung telah membuat perkampungan porak-poranda.

12. Nama-nama calon ketua perkumpulan ini telah tercatat dalam daftar.

Makna imbuhan ter- seperti dalam kalimat di atas ada pada kalimat …

A. Pengumuman itu tertempel di papan pengumuman.


B. Suara-suara kendaraan di jalan raya terdengar sampai di ruang ini.
C. Karena mengantuk, beberapa anggota rapat tertidur.
D. Pengunjung dapat tergelincir jika tidak hati- hati.
E. Beberapa gunung terlihat dari bukit ini.

13. Pihaknya sedang menyiapkan strategi mengolah sampah di dalam kota sehingga Jakarta tidak
penuh sampah lagi.

Kalimat yang polanya sama dengan kalimat di atas adalah …

A. Penanganan sampah lidak hanya dilakukan dengan menambah pengangkut sampah, tetapi
juga dengan menyediakan area penimbunan.
B. Kalangan pengusaha terpanggil untuk memberikan bantuan truk sampah sebagai wujud
persahabatan dengan Jakarta.
C. Peremajaan truk sampah sangat mendesak karena tidak sedikit truk sampah DKI yang sudah
tidak layak operasi.
D. Pemerintah DKI Jakarta akan menambah kontainer penimbunan sampah di sekitar rumah
warga.
E. Pemerintah akan menyediakan insinerator di berbagai wilayah agar biaya pengolahan
sampah dapat ditekan.

14. Kalimat yang tidak baku adalah …


A. Indonesia dengan penduduk 250 juta jiwa di mana mereka tinggal tersebar di pulau--pulau
Indonesia.
B. Kasus penebangan liar di daerah itu masih sering terjadi.
C. Pengadilan akan melaksanakan tugasnya untuk memberi hukuman setimpal pada para
pelanggar.
D. Interaksi antar kebudayaan dalam kelompok masyarakat dijalin tidak hanya meliputi suku
bangsa yang berbeda, melainkan juga antarperadaban di dunia.
E. Banyak juga dijumpai seni tari yang memiliki nilai-nilai keagamaan.

15. Yang termasuk kalimat baku adalah …


A. Habisnya hutan di wilayah hulu membuat air hujan tidak dapat diserap tanah sehingga
volume air yang melaju di permukaan meningkat.
B. Mempertahankan bahasa daerah juga sangat dibutuhkan peran orang tua terhadap anak
anaknya untuk membiasakan berkomunikasi dengan menggunakan bahasa daerahnya.
C. Sudah kerap dikemukakan, banjir datang bersamaan dengan menghilangnya wilayah-
wilayah resapan air berupa hutan lindung dan hutan tutupan di hulu-hulu sungai.
D. Tidak adanya kaitan halo matahari dengan prediksi gempa sehingga masyarakat tidak perlu
khawatir dan mengaitkannya dengan gempa.

4
alternatifa

E. Dari hari ke hari, unluk atas nama pertumbuhan ekonomi, untuk atas nama pertambahan
populasi, dan entah unluk atas nama apa lagi, kita justru terus menambah dosa-dosa
ekologis kita.

16. Senyuman dapat menstimulasi otak sehingga membuat pikiran lebih positif.

Makna kata bercetak miring di atas adalah …

A. Merangsang.
B. Mengalirkan.
C. Mengaktifkan.
D. Menghubungkan.
E. Membuat jaringan.

17. Meskipun Indonesia adalah negara dua musim, koleksi permainan baru yang memang mengikuti
negara asal produk ini, yakni London, tetap bisa dimainkan oleh anak-anak di tanah air.

Inti kalimat di atas adalah …

A. Indonesia negara dua musim.


B. Koleksi permainan mengikuti negara asal.
C. Koleksi permainan dimainkan anak-anak.
D. Koleksi permainan tetap bisa dimainkan.
E. London negara asal produk.

18. Kalimat yang mengandung kata yang bermakna idiomatis adalah …


A. lndustri batik belum berkembang secepat industri lain.
B. Potensi ekonomi rakyat ini perlu dikelola agar tumbuh sejalan dengan target nasional,
membangun ekonomi kreatif berbasis kerakyatan.
C. Bertempat di Pusat Kegiatan Mahasiswa, Mendikbud dan para wakil mahasiswa
mengadakan pembicaraan dari hati ke hati.
D. Ajang pameran lndocraft di Jakarta itu menyiratkan prospek cerah pengembangan industri
batik.
E. Ajang pameran 300 peserta yang memamerkan beragam produk batik ini menarik perhatian
lebih dari 15.000 pengunjung selama tiga hari.

19. Makna idiomatis terdapat dalam kalimat …


A. Untuk mempercepat pencapaian tujuan, kami mengambil jalan pintas lewat belakang
kampus.
B. Anak yang sering tinggal kelas itu dijuluki oleh teman-temannya si kerbau.
C. " Perempuan sendirian keluar malam--malam tak pantaslah!" teriak lbu.
D. Tokoh Gatotkaca berotot kawat dan bertulang besi.
E. Mata kita silau apabila menatap langsung si raja siang, matahari.

20. Melahirkan semangat Pancasila dapat diartikan sebagai keinginan untuk menegaskan diri
kembali pada komitmen ber-Pancasila.

Padanan yang tepat untuk kata komitmen dalam kalimat di atas adalah

A. Tekad.

5
alternatifa

B. Perjanjian.
C. Keinginan.
D. Kemauan.
E. Tanggung jawab.

6
alternatifa

UTUL UGM BAHASA INDONESIA 622


Bacaan berikut digunakan untuk menjawab soal nomor 1 sampai dengan nomor 3.

Dalam dua bulan berjalan, setidaknya ada peristiwa kebudayaan penting yang menempatkan candi
sebagai sumber "energi". Pada awal Maret 2014, sekelompok seniman mementaskan lakon "Shima:
Kembalinya Sang Legenda". Dalam pementasan pada tanggal 1- 2 Maret di Gedung Kesenian Jakarta (GKJ)
itu, sutradara Putut Budi Santosa melibatkan dua arkeolog, yakni Edy Sedyawati dan Inda C. Noerhadi.
Kemudian, dramaturgi pertunjukan berbentuk drama tari disusun berdasarkan temuan-temuan arkeologis
di Candi Dieng.

Dieng dipakai sebagai acuan riset mengingat keberadaan Ratu Shima sebagai raja negeri Kalingga
hanya ditemukan berdasarkan kabar para pencatat dari negeri Tiongkok. Candi Dieng menurut kajian
arkeologis memiliki kesamaan zaman dengan keberadaan Kalingga yang diperkirakan berasal dari 1.500
tahun yang lalu.

Kolaborasi antara sutradara sekaligus koreografer Putut Budi Santosa, penata musik Joko
Winarko, perancang busana Lucky Wijayanti, serta pendesain aksesori Terry W. Supit, dan para arkeolog
menghasilkan gambaran pertunjukan yang detail. Mereka bekerja tidak hanya dalam kerangka mengejar
nilai, tetapi juga memperhatikan bentuk-bentuk aksesori, busana, dan alat musik yang diperkirakan ada di
masa Kalingga. Tinggalan-tinggalan, seperti relief dan bentuk candi, dibaca ulang, lalu diinterpretasikan
untuk menemukan ragam hias busana dan aksesori yang mendekati aslinya.

1. Pernyataan yang tidak berhubungan dengan isi bacaan di atas adalah …


A. Kolaborasi antar seniman menghasilkan sebuah pertunjukan yang unik.
B. Pementasan drama tari yang mempertimbangkan temuan-temuan arkeologis pada sebuah
candi merupakan sesuatu yang relatif baru.
C. Candi Dieng digunakan sebagai sumber inspirasi dalam pementasan drama tari "Shima:
Kembalinya Sang Legenda".
D. Pementasan drama tari "Shima: Kembalinya Sang Legenda" merupakan hasil kerja sama antara
sutradara, seniman panggung, dan arkeolog.
E. Para arkeolog memberikan sumbangan kepada seniman dalam menafsirkan bentuk ragam hias
busana dan aksesori berdasarkan relief dan bentuk Candi Dieng.

2. Inti bacaan di atas adalah …


A. Candi Dieng digunakan sebagai sumber inspirasi dalam penyusunan drama tari "Shima:
Kembalinya Sang Legenda".
B. Pementasan drama tari "Shima: Kembalinya Sang Legenda" merupakan peristiwa kebudayaan
yang penting.
C. Kolaborasi antara seniman dan pakar arkeologi menghasilkan gambaran pertunjukan yang
lebih berkualitas.
D. Penempatan candi sebagai sumber penyusunan pertunjukan drama tari menghasilkan sebuah
pertunjukan yang menyegarkan.
E. Pementasan drama tari "Shima: Kembalinya Sang Legenda" disusun berdasarkan temuan-
temuan arkeologis di Candi Dieng.

3. Topik bacaan di atas adalah …


A. Pementasan drama tari "Shima: Kembalinya Sang Legenda".
B. Pemanfaatan candi sebagai salah satu sumber inspirasi dalam penyusunan drama tari.
C. Pementasan drama tari yang disusun dengan mempertimbangkan temuan-temuan arkeologis
pada sebuah candi.
alternatifa
D. Pertunjukan drama tari yang menandai peristiwa penting dalam kebudayaan.
E. Kolaborasi antara seniman dan pakar arkeologi dalam pertunjukan drama tari.

Penurunan populasi ikan di kawasan barat Indonesia, khususnya Laut Jawa dan Selat Malaka, disebabkan
oleh intensitas penangkapan yang tidak diimbangi dengan pengembangbiakan yang cukup. Pohon bakau
yang menjadi habitat alami ikan nyaris punah di wilayah itu sehingga mengurangi jumlah stok ikan secara
alami pula. Di sinilah pentingnya revitalisasi pesisir dengan konsep baru, yakni menjaga harmoni
ekosistem laut dengan aktivitas manusia. Laut seharusnya menjadi subjek yang dibutuhkan manusia
sehingga perlu dikelola dengan baik dan bijaksana.

4. Simpulan paragraf di atas adalah penurunan populasi ikan terjadi karena …


A. Penurunan populasi ikan terjadi karena tidak ada usaha revitalisasi pesisir.
B. Manusia tidak menjaga keberadaan pohon bakau sebagai habitat alami ikan.
C. Populasi ikan tidak berhubungan erat dengan intensitas penangkapan.
D. Manusia sebagai subjek terlalu berkuasa atas sumber daya laut.
E. Selama ini, laut dijadikan objek sehingga tidak tercipta harmoni ekosistem laut dengan aktivitas
manusia.

Dampak buruk mengonsumsi minuman soda berlebihan ternyata tidak dapat diremehkan. Gula dan asam
pada minuman bersoda dapat mengikis email gigi. Sementara itu, sodium benzoat yang ditemukan dalam
soda dapat mengurangi persediaan potasium dan bisa memicu asma. Banyak mengonsumsi minuman
ringan juga dapat meningkatkan risiko sindrom metabolisme, seperti diabetes dan gangguan hati.
Kandungan asam fosfat yang tinggi dalam soda memengaruhi batu ginjal dan masalah ginjal lainnya.

5. Pernyataan yang tidak berhubungan dengan isi paragraf di atas adalah …


A. Mengonsumsi minuman soda berlebihan memiliki dampak buruk.
B. Email gigi dapat terkikis karena minuman bersoda.
C. Banyak dampak serius akibat mengonsumsi minuman soda berlebih.
D. Minum minuman soda berlebih dapat menimbulkan beragam masalah.
E. Minum soda tidak memperburuk kesehatan.

Pemerintah bisa mengatasi masalah belum optimalnya manfaat pembangunan ekonomi dengan melakukan
redistribusi pendapatan melalui anggaran dan pajak. Di satu sisi, besarnya subsidi energi, misalnya, jelas
membuat upaya redistribusi sia-sia karena penerima subsidi justru kelompok kaya. Di sisi yang lain,
pemerintah harus menaikkan anggaran infrastruktur yang masih rendah untuk membuka kesempatan
kerja. Dengan demikian, pemerintah harus menerapkan pajak progresif yang tinggi untuk kepemilikan
properti dan mobil. Upaya ini ibarat sekali merengkuh dua tujuan tercapai: mengatasi kesenjangan
sekaligus mengurangi subsidi.

6. Ide pokok paragraf di atas adalah …


A. Redistribusi pendapatan melalui anggaran dan pajak.
B. Belum optimalnya pembangunan ekonomi.
C. Penerapan pajak progresif.
D. Upaya redistribusi.
E. Langkah mengurangi subsidi.

Faktor risiko yang berhubungan erat dengan kanker kolon atau kanker yang menyerang usus besar adalah
gaya hidup. Tak dapat dipungkiri bahwa kanker kolon merupakan penyebab kematian kedua di dunia.
Pencegahan kanker kolon dapat dilakukan dengan cara meningkatkan konsumsi serat makanan, termasuk
prebiotik dan probiotik (sinbiotik) yang dikenal masyarakat.

7. lsi pokok paragraf di atas adalah


A. Kanker kolon merupakan penyebab kematian kedua di dunia.
B. Kanker kolon yang diakibatkan oleh faktor gaya hidup dapat dicegah dengan cara
meningkatkan konsumsi serat makanan.
alternatifa
C. Susu fermentasi merupakan salah satu produk prebiotik dan probiotik (sinbiotik) yang dapat
mencegah risiko kanker kolon.
D. Meningkatkan konsumsi susu fermentasi merupakan cara yang dapat memperkecil risiko
terkena kanker kolon.
E. Kanker kolon yang merupakan penyebab kematian kedua di dunia dapat dicegah.

8. Penulisan unsur serapan yang tidak tepat terdapat dalam kalimat …


A. Desainer-desainer tersebut membuat koleksi furnitur yang bahan bakunya sisa-sisa letusan
Gunung Etna di Sisilia, Italia.
B. Survei pada 2012 menyebutkan bahwa 86 persen miliarder sukses karena usaha mereka
sendiri.
C. Hal tersebut membuat imajinasi anak menjadi lebih berkembang dan kreativitas mereka pun
terangsang.
D. Ketersediaan perangkat keras dengan harga terjangkau menjadi pemicu signifikan
perkembangan industri perangkat lunak.
E. Di dalam negeri, sangat banyak obyek wisata yang dapat dijelajahi.

9. Penulisan huruf kapital yang semuanya benar terdapat dalam kalimat …


A. Kepala Bidang Pensosbud KJRI Sydney mengunjungi sekolah di Australia guna mengajarkan
kesenian Indonesia, seperti angklung, tari Saman, dan tari Bali.
B. Saya tertarik pada isi artikel "Dampak Minuman Dengan Pemanis Buatan dan Pengawet Untuk
Tubuh Kita".
C. Lebih dari 2,5 juta wanita Indonesia menjadi Tenaga Kerja Wanita, padahal ancaman kekerasan
berada di depan mata.
D. Sistem Kapitalisme sebagai sistem yang diterapkan oleh kebanyakan Negara di dunia memiliki
cara pandang yang khas dan akan memengaruhi kebijakan yang dikeluarkan oleh Pemerintah.
E. Saat ini sudah sangat luas daratan Pedesaan Cemara Jaya yang tergerus akibat abrasi Pantai
Cemara.

10. Kata bentukan yang tidak tepat terdapat dalam kalimat …


A. Penambangan liar di Kabupaten Grobogan sangat memprihatinkan.
B. Koalisi bukan hal yang mudah untuk dilakukan karena membutuhkan penyelarasan visi, misi,
kepentingan, dan kompromi publik.
C. Model koalisi parpol untuk kemenangan pemilu selama ini adalah koalisi transaksional.
D. lndeks pembangunan manusia merupakan pengukuran perbandingan antara harapan hidup,
melek huruf, pendidikan, dan standar hidup untuk semua negara.
E. Pelibatan komunitas masyarakat menjadi bagian penting dalam usaha memberantas
vandalisme hingga tuntas.

11. Pemakaian tanda koma yang tepat terdapat pada kalimat…


A. Pertumbuhan apartemen yang menjamur, diikuti dengan permukiman padat di bawah garis
ambang luap air (gala) yang kumuh, merupakan gejala pertumbuhan kota yang tak terencana.
B. Jika Indonesia baru memiliki 15 hasil kebudayaan yang ditetapkan menjadi warisan budaya,
Jepang dan Korea Selatan, sudah memiliki masing-masing 43 dan 37 buah jenis kebudayaan.
C. Menurut pakar diet klinis, Emilia E. Achmadi, M.S., jika teh berpemanis buatan diminum satu
kali saja, efek negatifnya tidak akan signifikan.
D. "Kita harus bisa membuktikan bahwa tenun memang milik masyarakat, dan memiliki
keterkaitan dengan kehidupan mereka", jelas Wamendikbud Bidang Kebudayaan.
E. Oleh karena itu, data-data yang dibawa seperti sejarah, hasil penelitian, dan pemetaan, menjadi
unsur yang kuat agar tenun NTT terpilih sebagai warisan budaya dunia.
alternatifa
Kelompok pemuda-pemuda yang tergabung dalam gerakan “Revolusi Putih” membagi-bagikan 10.000
kotak susu di Bundaran Hotel Indonesia, Jakarta. Tidak lupa dalam kesempatan tersebut mereka mengajak
tokoh- tokoh superhero “Mas Garuda" dan "Arya Bima” untuk terlibat. Anak-anak yang memadati wilayah
itu pun sangat antusias menyaksikan kegiatan-kegiatan itu.

12. Pada paragraf di atas terdapat penggunaan kata ulang yang tidak tepat , yaitu kata
A. pemuda-pemuda
B. membagi-bagikan
C. tokoh-tokoh
D. anak-anak
E. kegiatan-kegiatan

13. Yang merupakan kalimat baku adalah …


A. Gunung Rinjani merupakan gunung yang terletak di Nusa Tenggara Barat.
B. Banyak pulau-pulau menjadi habitat satwa komodo di pulau Komodo.
C. Berdasarkan keterangan direktur perusahaan industri tekstil itu tidak memenuhi syarat.
D. Dengan terungkapnya kasus tindak korupsi ini diharapkan dapat ditangani secepatnya.
E. Sebagai sebuah objek wisata, Gunung Bromo memiliki daya tarik tersendiri.

Untuk tahun 2014, anggaran negara yang mencapai Rp150 triliun sebagian besar akan dialokasikan untuk
revitalisasi industri pertahanan.

14. Kalimat yang memiliki pola yang sama dengan kalimat di atas adalah …
A. Alat yang sudah tidak berfungsi pun, sebagaimana dipaparkan Military Balance, ada yang masih
dimasukkan anggaran pemeliharaannya.
B. Ada empat belas upaya yang telah dilakukan BNPB untuk menanggulangi dampak bencana
nasional.
C. Presiden meminta Komisi Pemberantasan Korupsi (KPK) memantau proses pengadaan barang
dan jasa, yang tergolong rawan korupsi, termasuk di tubuh TNI.
D. Pada salah satu industri strategis, pemesanan barang dan jasa malah dikurangi, bahkan
dibatasi.
E. Berdasarkan kajian ini, perebutan posisi di komisi ekonomi yang menduduki peringkat
tertinggi di beberapa di antaranya direspons secara berlebihan oleh petinggi partai.

Kaum miskin dan menengah yang ada di sejumlah negara Asia Tenggara dan Pasifik Barat paling merasakan
dampak polusi udara pada tahun 2012 dengan 3,3 juta kematian akibat polusi di dalam rumah dan 2,6 juta
lainnya akibat polusi di luar rumah.

15. Kalimat tersebut merupakan perluasan dari kalimat inti …


A. Kaum miskin dan menengah merasakan dampak polusi udara. .
B. Kaum miskin dan menengah ada di sejumlah negara.
C. Kaum miskin dan menengah mengalami kematian.
D. Ada 3,3 juta orang yang mengalami kematian akibat polusi udara.
E. Polusi udara menyebabkan kematian

Masih belum diakuinya aktivitas ekonomi kreatif oleh perbankan membuat mereka sulit mendapatkan
dukungan finansial. Minimnya modal itu telah memangkas kreativitas mereka.

16. Makna kata memangkas pada kalimat di atas adalah …


A. Memotong
B. Menurunkan
C. Membatasi
D. Memagari
E. Menghilangkan
alternatifa
17. Kalimat yang mengandung makna idiomatis adalah …
A. Tangan kanannya terkilir sehingga tidak dapat mengangkat barang-barang berat.
B. Karena dijadikan kambing hitam, ia menempuh jalur hukum.
C. Meja hijau ini sudah lama tidak dicat ulang sehingga warnanya sudah tampak pudar.
D. Kepalanya keras, sekeras batu hitam.
E. Karena suatu penyakit, tangannya selalu dingin.

18. Yang merupakan kalimat baku adalah …


A. Pengawetan telur dapat dilakukan dengan beberapa cara, antara lain pendinginan;
membungkus kering; proses pelapisan dengan minyak; dan mencelupkan dalam berbagai
cairan.
B. Telur jika hanya sekadar dikonsumsi dalam keadaan segar saja tidak mampu disimpan dalam
waktu yang terlalu lama.
C. Telur-telur diperlakukan dengan baik supaya awet dan daya simpannya dapat dipertahankan
dalam waktu yang lebih lama.
D. Untuk menjaga kesegaran dan kualitas isinya, telur memerlukan Teknik penanganan yang
tepat.
E. Secara umum prinsip pengawetan telur adalah untuk tujuan agar mencegah masuknya bakteri
pembusuk dalam telur yang dapat merusak telur.

19. Diskursus budaya tidak hanya berkaitan dengan sektor seni dan adat-istiadat, tetapi juga dengan
perilaku dan cara berpikir.

Padanan makna yang tidak tepat untuk kata diskursus dalam kalimat di atas adalah …
A. Pertukaran ide
B. Bahasan
C. Wacana
D. Persebaran
E. Rasionalitas

20. Kata yang mengalami perubahan makna menyempit terdapat dalam kalimat …
A. Para tokoh mengajak masyarakat untuk menggunakan hak pilihnya dengan cerdas dan tanpa
tekanan agar kursi DPR terwakili secara benar.
B. Keutuhan fungsi UN terkait dengan pemetaan indeks kompetensi, kelulusan seleksi, · dan
perbaikan terhadap infrastruktur sekolah.
C. Para sarjana lingkungan berkumpul di Kanada untuk mengikuti konferensi internasional
tentang perubahan iklim.
D. Kapal-kapal Australia berlayar sejak beberapa hari yang lalu mencari kotak hitam pesawat
MH370.
E. “Maaf, apakah Bapak tahu gedung induk tempat seminar berlangsung?"
alternatifa

UTUL UGM 2016 Bahasa Indonesia 372

Bacaan Berikut digunakan untuk menjawab soal nomor 1 sampai dengan 4

Kemajuan teknologi membuai siapa saja yang memanfaatkannya. Tak jarang kemajuan teknologi
membuat orang kecanduan dan tidak bisa lepas dari perangkat digital dalam hubungan sosial. Ada sindrom
yang membuat orang-orang takut dan gelisah jika berjauhan dengan perangkat teknologi dan kehidupan
di media sosial, yaitu Fear of Missing Out (FOMO). Lalu, mengapa fenomena FOMO ini lebih banyak terjadi
pada perempuan?

FOMO adalah sebuah fenomena mengenai seseorang yang merasa takut dan gelisah jika tidak
mengikuti obrolan dan isu-isu di media sosial. FOMO disebut-sebut sebagai pemicu terjadinya nomophobia
atau ketakutan jika berjauhan dengan telepon genggam. Beberapa jenis gangguan kejiwaan tersebut
disebabkan oleh fenomena FOMO. FOMO telah diteliti secara mendalam dan dipublikasikan di jurnal
Computers in Human Behaviour pada tahun 2013 silam.

Faktor yang membuat FOMO semakin fenomenal adalah tingkat pengguna media sosial yang
sangat tinggi saat ini, terutama di tanah air. Hasil penelitian tentang FOMO menunjukkan bahwa sampel
objek di bawah usia 30 tahun memiliki kecenderungan tertinggi mengalami FOMO. Selain itu, uniknya,
perempuan disebut lebih banyak mengalami FOMO daripada laki-laki. (Sumber: Kartini, September 2015,
hlm 68)

1. Pernyataan yang tidak berhubungan dengan bacaan di atas adalah …


A. FOMO lebih banyak dialami oleh perempuan
B. FOMO adalah penyakit yang sama dengan nomophobia
C. Pengguna media sosial yang sangat tinggi membuat FOMO semakin fenomenal
D. Kemajuan teknologi membuat orang kecanduan menggunakan perangkat digital dalam
hubungan sosial
E. Kecenderungan tertinggi FOMO dialami oleh orang di bawah usia 30 tahun

2. Maksud “fenomenal” dalam bacaan di atas adalah …


A. Ajaib
B. Menghebohkan
C. Luar biasa
D. Menggejala
E. Bombastis

3. Simpulan bacaan di atas adalah …


A. FOMO lebih banyak dialami oleh perempuan yang usianya di bawah 30 tahun
B. Nomophobia dipicu oleh FOMO
C. Kemajuan teknologi memunculkan sindrom baru yang menjadi perhatian khusus
D. FOMO adalah sindrom baru yang menjadi perhatian khusus
E. Media sosial membuat orang cenderung mengalami nomophobia dan FOMO

4. Pertanyaan yang jawabannya terdapat dalam bacaan di atas adalah …


A. Mengapa FOMO menjadi pemicu terjadinya nomophobia?
B. Di mana dilakukan penelitian terkait FOMO dan nomophobia?
C. Kapan FOMO mulai fenomenal?
D. Mengapa orang takut dan gelisah ketika tidak menggenggam telepon genggam?
alternatifa
E. Apa hasil penelitian yang ada dalam jurnal Computers in Human Behaviour?

5. Bentuk serapan yang benar terdapat dalam kalimat …


A. Perluasan pembangunan sampai ke wilayah pedalam dilakukan dengan tetap menempatkan
warga sekitar sebagai subyek kemajuan
B. Pemerintah memiliki program bantuan langsung tunai untuk menurunkan angka kemiskinan,
tetapi sebenarnya akan lebih bagus apabila dana tersebut dimanfaatkan untuk memacu
kreatifitas masyarakat
C. Aksi pendidikan berwawasan meliputi usaha-usaha melokalisir bidang-bidang yang berpotensi
berhasil mengarahkan warga negara di bidang hukum.
D. Pemerintah diharapkan dapat memberikan respons positif terhadap gagasan bahwa
membentuk negara sebagai sarana membangun jiwa
E. Alih-alih bersikap konfrontatif, dia justru menunjukkan sikap kooperatip dalam tindak pidana
yang didakwakan kepadanya

6. Pemakaian tanda baca yang tepat dalam kalimat …


A. Kita sering mendengar upaya membangun kota cerdas, kota berwawasan lingkungan, kota
hijau hingga kota kreatif
B. Indonesia berkepentingan menumbuhkan pelaku wirausaha karena berciri inovatif, berani
mengambil risiko, berorientasi ke depan, berjiwa merdeka
C. Karakteristik pembangunan kota ―hampir tidak dapat dipungkiri― mengandaikan pemerintah
kota yang paling menentukan dan paling bertanggung jawab
D. Pekerjaan sebagai petani ditinggalkannya jumlah penduduk bertambah cepat, sumber daya
desa menyusut, dan kemakmuran di kota meningkat
E. Besarnya jumlah orang desa tidak berketerampilan khusus yang berpindah ke kota
menimbulkan masalah sosial; pengangguran bertambah, kriminalitas meningkat

7. Pemakaian huruf kapital yang tepat terdapat dalam kalimat …


A. Bukit yang sekarang dikenal sebagai Punthuk Setumbu terletak sekitar 4 Km di sebelah barat
Candi Borobudur.
B. Raffles adalah seorang Gubernur pemerintahan kolonial yang bertugas di Jawa pada tahun
1811-1815.
C. Bali memiliki banyak tempat wisata, salah satunya Air Terjun Nungnung di wilayah Badung,
Dusun Nungnung, Desa Plaga, Kecamatan Petang.
D. Dari aspek biologi, kawasan Danau Toba memiliki keunikan tersendiri, yaitu adanya Pohon
Hanara.
E. Beberapa Objek Wisata yang dapat dikunjungi di sekitar Pantai Tanjung Kelayang antara lain
adalah Pulau Burung Garuda, Pulau Batu berlayar, dan Pulau Lengkuas

8. Makna pengulangan kejar-kejaran dalam kalimat Di kanan kiri kapal yang saya tumpangi terdapat
ikan lumba-lumba berenang kejar-kejaran beradu cepat juga terdapat dalam kalimat …
A. Antrean para calon penumpang KA khusus liburan panjang akhir pekan berurut-urutan sampai
gerbang stasiun.
B. Proses belajar mengajar di sekolah itu berlangsung dengan disiplin tinggi dan penuh semangat
kejuangan.
C. Masa studinya yang panjang di salah satu negara di Semenanjung Arabia itu menyebabkan gaya
bicaranya kearab-araban.
D. Setinggi-tingginya harga lukisan akhir abad ke-19 itu tidak akan mencapai tiga kali lipat
tawaran harga lelang pertama.
E. Meskipun hanya melakukan latihan rutin menjelang pertandingan, atlet bela diri yang tengah
naik daun itu terlihat pukul-memukul dengan mitra tandingnya.
alternatifa
9. Penggunaan kata berimbuhan yang tidak tepat terdapat dalam kalimat …
A. Untuk menata kembali koleksi dan administrasi, dibutuhkan program kegiatan penyiangan
B. Dalam kegiatan tersebut dilakukan penghitungan koleksi untuk mengetahui keseluruhan
koleksi yang ada
C. Selain itu, dilakukan pula perbaikan atas koleksi yang rusak agar dapat dimanfaatkan lagi
D. Indonesia membutuhkan revolusi kebijakan investasi untuk mendukung strategi penumbuhan
ekonomi
E. Sudah ada upaya penanaman, tetapi hasilnya belum sesuai dengan harapan.

10. Kalimat yang tidak baku adalah …


A. Forum ekonomi dunia menerbitkan peta risiko yang dihadapi dunia berdasarkan indikator
ekonomi, lingkungan sosial, geopolitik, dan teknologi.
B. Perekonomian dunia sudah beberapa waktu hadapi kondisi yang sangat tidak menentu.
C. Negara-negara pemilik hutan tropis terbesar terus mengubah hutan mereka menjadi lahan
peternakan, area tambang batu bara, dan perkebunan kelapa sawit.
D. Sesuai dengan siklus biologinya, pohon akan menghisap dan mengubah karbon menjadi
biomassa.
E. Dunia mungkin tidak akan dapat lagi bergantung pada hamparan pohon dan tumbuhan raksasa.

11. Selain antisipasi terhadap situasi global yang belum akan membaik dua tahun ke depan, langkah
mempertahankan suku bunga The Fed paling tidak akan mengurangi potensi gejolak di pasar uang
dan bursa saham negara berkembang.

Inti kalimat di atas adalah …


A. Perlu antisipasi terhadap situasi global
B. Situasi global belum akan membaik
C. Langkah mempertahankan suku bunga
D. Langkah mengurangi potensi gejolak
E. Bursa saham negara berkembang

12. Hipotesis mengenai asal mula Gunung Nglanggeran sebagai pusat erupsi purba dimunculkan karena
pertimbangan morfologinya yang kontras dengan sekitarnya padahal masih dalam satu formasi
dengan batuan yang sama.

Inti kalimat di atas adalah …


A. Hipotesis dimunculkan
B. Asal mula Gunung Nglanggeran
C. Pusat erupsi purba dimunculkan
D. Gunung Nglanggeran satu formasi dengan batuan yang sama
E. Asal mula Gunung Nglaggeran kontras

13. Kalimat yang baku adalah …


A. Berdasarkan keterangan Direktur perusahaan menerangkan bahwa pihaknya tidak tahu
adanya pemalsuan terhadap produk mereka.
B. Penjelasan daripada karyawan yang bekerja di perusahaan itu sangat diperlukan.
C. Konferensi pers akan dilakukan agar supaya permasalahan pemalsuan produk ini dapat
menemukan titik terang.
D. Mereka mendatangi perusahaan itu hanya sekedar untuk mengonfirmasi isu yang beredar saja.
E. Pada akhirnya, semua pihak mendapatkan bagian yang sesuai dengan yang sudah mereka
usahakan
alternatifa
14. Tingkat suku bunga yang tinggi akan mengganggu daya saing dunia usaha di pasar regional dan
internasional pada era MEA yang sudah dimulai.

Kalimat di atas mempunyai pola yang sama dengan …


A. Dalam mendorong investasi swasta, tantangan berada pada kemampuan pemerintah
mewujudkan janji dalam paket stimulus perekonomian
B. Rekomendasi Bank Dunia adalah memperkuat investasi swasta karena perluasan fiskal tidak
akan dapat mendorong pertumbuhan ekonomi 5%
C. Kelompok bank pemerintah menguasai lebih dari separuh pasar industri perbankan di setiap
provinsi sejak runtuhnya bank-bank swasta pada beberapa tahun yang lalu
D. Pemerintah mengeluarkan Surat Utang Negara (SUN) untuk menambah modal bank nasional
yang kekurangan modal dan mengatasi kredit yang bermasalah
E. Cara paling mudah untuk menurunkan tingkat suku bunga adalah meningkatkan efisiensi dan
produktivitas bank-bank pemerintah, baik milik pemerintah pusat maupun pemerintah daerah

15. Orang terpesona, terpana, dan mengira bahwa dia sedang berada di suatu tempat yang sangat eksotis
ketika gerhana matahari penuh.

Padanan kata yang tepat untuk kata eksotis dalam kalimat di atas adalah …
A. Indah
B. Asing
C. Seksi
D. Merangsang
E. Menarik

16. Kata bercetak miring yang mengalami perluasan makna terdapat dalam kalimat …
A. Meskipun tahun 2016 perekonomian Indonesia diperkirakan masih suram, optimisme
terhadap Indonesia sebagai kekuatan ekonomi dan pasar besar masih kuat
B. Karirnya dalam dunia hiburan tidak redup walaupun beberapa kali isu tidak sedap menerpanya
C. Di muka sidang pengadilan, terdakwa membeberkan pihak-pihak yang menerima amplop dan
tindak pidana korupsi yang didakwakan kepadanya
D. Ketimpangan kepemilikan lahan berakibat pada ketimpangan kemakmuran, terutama bagi
rakyat yang menggantungkan hidupnya pada penguasaan tanah
E. Dalam praktik sehari-hari, fungsi tanah semakin jauh dari isi Pasal 6 Undang-Undang Nomor 5
tahun 1960 tentang Pokok-Pokok Agraria

17. (1) Persepsi masyarakat yang bersifat positif terhadap cendana tersebut dapat menjadi modal sosial
bagi upaya pelestarian cendana.
(2) Meskipun dilakukan secara tradisional, tingkat keberhasilannya cukup menggembirakan,
terlebih lagi jika dilakukan peningkatan penguasaan teknik budidaya sesuai dengan standar
petunjuk teknis budidaya cendana.
(3) Di antara pihak-pihak tersebut, terdapat masyarakat yang secara swadaya dan mandiri sudah
melakukan upaya pelestarian cendana.
(4) Dalam upaya pelestarian tersebut, masyarakat masih menggunakan teknik budidaya dan
pemeliharaan cendana secara tradisional.
(5) Penurunan populasi cendana yang terjadi di Nusa Tenggara Timur (NTT) beberapa dekade
terakhir menyebabkan berbagai pihak mengupayakan aksi pelestarian cendana.

Urutan yang tepat agar kalimat-kalimat di atas menjadi paragraf yang padu adalah …
alternatifa
A. (1), (2). (3), (4), (5)
B. (1), (4), (2). (5), (3)
C. (5), (4). (1), (3), (2)
D. (5), (3). (4), (2), (1)
E. (5), (4), (2), (3), (1)

18. Para peneliti mempelajari tulang dan gigi dari 904 fosil mamalia berplasenta untuk mengukur
perbedaan anatomi dan antarspesies. Informasi yang didapat kemudian digunakan untuk
memperbarui pohon silsilah 177 spesies Eutheria―kelompok mamalia yang lebih dekat
kekerabatannya dengan manusia daripada dengan kanguru. Sebanyak 94 fosil diantaranya berasal
dari era Paleosen. Ini adalah pohon silsilah terbesar yang menggambarkan mamalia dari era Paleosen.
Silsilah yang dianalisis dengan pembagian waktu mulai 140 juta tahun lalu hingga hari ini tersebut
menunjukkan perubahan keragaman spesies mamalia.

Ide pokok paragraf di atas adalah …


A. Fosil mamalia
B. Asal fosil mamalia
C. Silsilah mamalia
D. Keragaman spesies mamalia
E. Perubahan spesies mamalia

19. Pemanasan global terjadi ketika suhu rata-rata atmosfer, laut, dan daratan meningkat. Hal itu
disebabkan oleh menumpuknya gas rumah kaca. Gas ini akan menjadi isolator yang menahan lebih
banyak panas matahari yang dipancarkan ke bumi. Memahami berapa banyak metana yang
dipancarkan ke bumi dan memahami berapa banyak metana yang dipancarkan ke atmosfer dari
semua sumber akan membantu para ilmuwan untuk memperhitungkan jumlah gas rumah kaca
global dan mengambil langkah-langkah untuk mengurangi dampaknya.

Simpulan paragraf di atas adalah …


A. Suhu rata-rata atmosfer, laut, dan daratan meningkat
B. Tumpukan karbon dioksida, metana, dan dinitrogen oksida mengumpul di atmosfer
C. Gas rumah kaca akan menghalangi sinar matahari ke bumi
D. Pemanasan global muncul karena peningkatan rerata suhu atmosfer
E. Jumlah gas rumah kaca dapat dihitung dan direduksi

20. Saturnus awalnya memiliki beberapa ring purba yang lebih banyak mengandung partikel daripada
yang mereka punya saat ini. Satelit-satelit Saturnus terbentuk dari partikel-partikel yang saling
bergabung. Pada tahap akhir formasi, beberapa satelit kecil terbentuk dalam orbit berdekatan. Data
yang diperoleh dari wahana Cassini mengindikasikan bahwa satelit-satelit kecil yang mengorbit di
sisi luar sistem cincin utama mempunyai inti padat.

Pernyataan yang tidak dikemukakan dalam paragraf di atas adalah….


A. Jumlah partikel ring purba berkurang
B. Partikel-partikel saling berhubungan
C. Temuan itu dapat mengungkap sistem satelit lain
D. Letak orbit satelit kecil tidak berjauhan
E. Beberapa satelit kecil tidak berada dalam cincin
alternatifa

UTUL UGM BAHASA INDONESIA 2017 748

1. Karena luas lahan sudah sangat terbatas, ekstensifikasi lahan tidak lagi menjadi pilihan dalam
upaya meningkatkan kualitas hasil pertanian di Pulau Jawa.

Padanan kata yang tepat untuk kata ekstensifikasi dalam kalimat di atas adalah …
A. Peluasan
B. Perluasan
C. Meluaskan
D. Memperluas
E. Keluasan

2. Kata tercetak miring dalam kalimat berikut mengandung perubahan makna, kecuall ...
A. Panitia ujian penerimaan pegawai negeri sipil itu menolak menerima uang pelicin dari
seorang calon peserta.
B. Segala urusan sebaiknya ditempuh dengan prosedur yang wajar, tidak perlu lewat jalan
belakang.
C. Meja hijau tengah menanti para politisi dan anggota dewan yang terbukti korup.
D. Meskipun baru pertengahan bulan, gaji kami sudah habis tak tersisa.
E. Selesai menjadi anggota legislatif, dia hanya bergaul dengan komunitas kelas atas.

3. Perangkat permainan harus bisa menjembatani kemampuan atlet olahraga elektronik untuk
memberikan perintah ke komputer dalam hitungan waktu sepersekian detik tanpa mereset.

Kalimat di atas merupakan perluasan dari kalimat inti…


A. Kemampuan atlet memberikan perintah.
B. Perangkat permainan memberikan perintah.
C. Kemampuan atlet memberikan perintah.
D. Perintah ke komputer sepersekian detik.
E. Perangkat menjembatani kemampuan.

4. Kalimat yang baku adalah ...


A. Direktur rumah sakit swasta itu mengakui, masih longgarnya keamanan di rumah sakit,
untuk itu pihaknya segera mengirim surat kepada para kepala bagian keamanan itu dan
meningkatkan keamanan.
B. Sebagai contoh, terkadang rumah sakit tidak disiplin dengan aturan seputar menjenguk
pasien, padahal, memerlukan aturan tersebut agar keluar masuknya pengunjung lebih
terkontrol dan pasien dapat beristirahat.
C. Kelonggaran tersebut terjadi karena pihak rumah sakit kadang-kadang khawatir dikatakan
kurang manusiawi terhadap pasien dan keluarganya.
D. Dia mengatakan sulit menertibkan seluruh rumah sakit agar berdisiplin menegakkan
aturan, untuk itu dibutuhkan kerja sama yang baik antara rumah sakit dan pasien.
E. Terkait keamanan, pemerintah memberikan panduan, tetapi sistem keamanan di rumah
sakit didesain dan dikembangkan oleh rumah sakit sesuai kebutuhannya serta kondisinya
masing-masing.

1
alternatifa

5. Sosialisasi perlu digalakkan demi kesuksesan kebijakan ini.

Kalimat yang polanya sama dengan kalimat di atas adalah ...


A. Seberapa optimal implementasinya di lapangan.
B. Pemerintah harus terus mengontrol program tersebut.
C. Toko modern sudah saatnya menjalankan imbauan plastik berbayar ini.
D. Budaya ini bisa dimulai dengan membawa tas sendiri saat berbelanja.
E. Langkah kecil ini akan didukung dengan kebijakan yang relevan.

6. Kalimat yang tidak baku adalah …


A. Pulau ini terdapat sangat banyak sumber mata air panas alam.
B. Sejumlah kolam itu merupakan peninggalan dari zaman penjajahan Jepang.
C. Kualitas air panasnya, sangat alami karena sumbernya tak jauh dari kolam-kolam air panas
itu.
D. Kandungan belerangnya yang tinggi membuat pemandian ini menjadi sarana pengobatan.
E. Umumnya orang menikmati suasana di pemandian air panas ini tak lebih dari satu jam.

7. Penulisan kata bercetak miring yang tidak mengikuti pedoman ejaan yang benar adalah ...
A. Di Indonesia praktek seperti itu baru diterapkan secara tumpang sari.
B. Deskripsi produk dapat menjadi alasan pengunjung membeli (atau tidak) di toko.
C. Elektronika sangat dibutuhkan karena hampir seluruh teknologi membutuhkan komponen
elektronik.
D. Pemerintah meluncurkan sistem informasi kesempatan kerja.
E. Putaran roda kereta api telah berpuluh tahun menemani Sopiah mencari rezeki di ibu kota.

8. Penulisan gabungan kata dalam kalimat berikut benar, kecuali …


A. Mengindonesiakan sebuah karya sastra asing bukanlah hal mudah karena harus memahami
ke-dua budaya sekaligus.
B. Pemimpin hanya menggarisbawahi apa yang sudah disampaikan oleh ketua adat pada
masyarakat.
C. Tanah longsor itu telah meluluh lantakkan rumah-rumah penduduk yang berada di bawah
tebing tersebut.
D. Kantor bupati itu kini dialihfungsikan sementara untuk menampung korban bencana banjir
di kabupaten kami.
E. Para koruptor itu bertepuk tangan setelah tuduhan jaksa penuntut umum dinyatakan kurang
mempunyai bukti yang kuat.

9. Buah-buahan dapat menjamin kesehatan tetap optimal dalam jangka panjang.

Kata ulang yang maknanya sama dengan kata buah--buahan dalam kalimat di atas adalah ...
A. Rumah-rumahan yang dijual di toko itu sangat unik karena dibuat dari barang-barang bekas.
B. Dalam permainan mobil-mobilan itu, anak-anak dapat berlomba merancang kendaraan.
C. Sungai-sungai yang sempat tercemar limbah pabrik kini sudah kembali bersih seperti
semula.
D. Di ruang itu kursi-kursi berserakan tak beraturan karena setelah digunakan tidak
dikembalikan lagi ke posisi semula.

2
alternatifa

E. Longsor di daerah tersebut terjadi karena pohon-pohonan ditebangi dan tidak diganti
dengan tanaman baru.

10. Bentukan kata yang benar terdapat dalam kalimat ...


A. Kestabilan negara-negara Timur Tengah sebagai pensuplai minyak dunia harus dijaga.
B. Semua warga negara Indonesia harus mentaati peraturan dan hukum yang berlaku tanpa
pandang bulu.
C. Perancang mode itu mengkombinasikan unsur modern dan lokal-tradisional pada busana
rancangannya.
D. Mahasiswa dipersilakan mengkritik pemerintah sebagai wujud kebebasan berpendapat yang
dijamin oleh undang-undang.
E. Orang itu mengontrakan rumahnya untuk mahasiswa asing yang sedang belajar bahasa dan
budaya Indonesia.

3
alternatifa

UTUL UGM BAHASA INDONESIA 2017 823

Bacaan ini dipergunakan untuk menjawab soal nomor 1-3.

Dinas Perhubungan Gunungkidul telah melakukan kajian terhadap penataan angkutan di daerah
tersebut. Armada angkutan perkotaan yang ada ternyata menambah beban konsumen, terutama yang
berasal dari daerah-daerah sehingga angkutan perkotaan (angkot) akan dilebur menjadi satu dalam
angkutan pedesaan (angkudes). Para penumpang yang berasal dari desa terpaksa berganti angkutan
untuk sampai ke kota, baik pusat pemerintahan kabupaten, pasar, maupun tempat lainnya sehingga
menambah biaya. Di samping itu, jumlah kendaraan, khususnya angkudes dan angkot, juga semakin
menyusut karena dijual ke luar daerah atau tidak diperpanjang izin trayeknya akibat semakin sepinya
penumpang.

Sumber. Kedaulatan Rakyat, 11 Februari 2017

1. Ide pokok bacaan di atas adalah ....


A. Hasil kajian terhadap penataan angkutan di Gunungkidul
B. Penyatuan angkot dan angkudes
C. Alasan keberatan penumpang angkutan
D. Menyusutnya jumlah angkutan di Gunungkidul
E. Sepinya penumpang angkutan di Gunungkidul

2. Inti bacaan di atas adalah ...


A. Dinas Perhubungan Gunungkidul telah melakukan kajian terhadap moda transportasi di
wilayah tersebut.
B. Kajian angkutan di Gunungkidul menyimpulkan perlunya penataan angkutan di daerah
tersebut.
C. Pemerintah Daerah Gunungkidul akan melakukan peleburan angkutan perkotaan dan
angkutan pedesaan.
D. Moda transportasi di Gunungkidul tidak cukup efektif.
E. Terjadi penyusutan jumlah angkutan yang cukup signifikan.

3. Pemyataan yang tidak berhubungan dengan isi bacaan di alas adalah ...
A. Angkutan perkotaan ternyata menambah beban konsumen.
B. Penumpang dari desa harus naik angkutan lebih dari satu kali untuk menuju pusat
pemerintahan.
C. Biaya perpanjangan trayek yang mahal membuat pemilik angkutan tidak memperpanjang
izin trayeknya.
D. Terjadi perubahan jumlah kendaraan di Gunungkidul.
E. Penumpang semakin berkurang sehingga banyak angkutan yang tidak beroperasi.

4. Penggunaan tanda baca koma (,) yang tepat terdapat dalam kalimat...
A. Beberapa mahasiswa yang lolos tes dan memenuhi persyaratan, akan mendapat beasiswa
dari universitas terkenal di Tokyo
B. Para mahasiswa yang akan mengikuti tes kemampuan bahasa Jepang secara gratis,
diharapkan mendaftarkan diri di ruang kantor

1
alternatifa

C. Para ahli matematika merasa kagum, ketika mengetahui perhitungan lebah yang sangat
cermat.
D. Seorang pedagang di pasar mengatakan, sambil melayani pelanggannya, bahwa kenaikan
harga cabai dan bawang merah sudah berlangsung selama tiga tahun.
E. Para penyelenggara negara yang dalam batas waktu tertentu tidak melaporkan harta
kekayaannya ke KPK, akan dikenakan sanksi yang berupa denda atau hukuman

5. Penulisan unsur serapan yang tidak sesuai dengan pedoman ejaan terdapat dalam kalimat ...
A. Duduk di bawah pohon rindang dapai membuat hati dan pikiran menjadi lebih lenang
sehingga stress pun hilang
B. Cara tersebut cukup efektif bila dilakukan oleh perempuan yang menderita gangguan
kecemasan.
C. Ragam aktivitas di luar ruang berpengaruh besar terhadap para remaja.
D. Untuk mendapatkan sensasi rileks yang optimal, segala macam gawai sebaiknya
ditinggalkan,
E. Jurnalis sains, Florence Williams, melaporkan bahwa beristirahat di bawah pohon rindang
sangat bermanfaat.

6. Penulisan kata yang tidak sesuai dengan pedoman ejaan terdapat dalam kalimat …
A. Buah dan sayuran nonorganik tidak selalu berarti tidak sehat.
B. Puting beliung telah memporak porandakan kawasan perumahan yang baru saja dibangun.
C. Buku ini memberi tahu para pembaca tentang pemikiran brilian sang proklamator.
D. Rompi antipeluru ini selalu mereka pakai ketika melakukan operasi.
E. la dapat menyelesaikan tugas itu dengan baik karena jalan pikirannya sangat sistematis.

7. Gabungan kata bercetak miring yang bukan kata majemuk terdapat dalam kalimat …
A. Kurikulum terbaru mengamanatkan agar setiap mata pelajaran di jenjang pendidikan dasar
sampai sekolah tingkat atas mengandung pendidikan karakter
B. Kapal-kapal modern juga didorong mematuhi aturan sandar, tidak melewati garis batas
zona kapal tradisional.
C. Pameran pendidikan dan peralatan sekolah paling bergengsi di Indonesia diselenggarakan
oleh pemerintah bersama universitas terkenal.
D. Pembangunan rumah susun model linear menunjukkan kepedulian pemerintah terhadap
kebutuhan hunian warga ibu kota.
E. Duta besar luar biasa dan berkuasa penuh adalah pejabat diplomatik yang ditugaskan ke
negara lain untuk mewakili negara.

8. Pemakaian kata berimbuhan yang tidak tepat terdapat dalam kalimat ...
A. Kekeliruan penyimpanan memori di otak bisa terjadi setelah mendengar atau membaca
banyak informasi.
B. Pemecahan ombak untuk mengurangi abrasi dapat dilakukan dengan beberapa cara.
C. Sebuah organisasi memberikan pemahaman yang bagus tentang bakau kepada Kelompok
Cinta Lingkungan.
D. Penanaman pohon jati dilakukan di lahan seluas dua hektare.
E. Penambahan jumlah warga yang dengan sadar ikut menjaga ekosistem cukup besar.

9. Kalimat berikut ini tidak baku, kecuali ...

2
alternatifa

A. Dari pengamatan di lapangan menunjukkan bahwa produk Indonesia terancam


keterpurukan dengan adanya kebijakan pasar bebas sejak beberapa tahun yang lalu.
B. Dengan berlakunya perjanjian tersebut, maka diharapkan Departemen Perindustrian perlu
memperjuangkan dengan sungguh-sungguh penundaan 228 item yang bebas bea masuknya
ke lndonesia.
C. Bagi mereka yang kini duduk di Kementerian Perdagangan sebaiknya mulai memikirkan
dampak implementasi perjanjian perdagangan bebas terhadap UKM yang ada di Indonesia.
D. Dari hasil penelitian terbukti bahwa investasi untuk efisiensi energi di lndonesia selama 10
tahun memerlukan dana 9 miliar dolar AS.
E. Kepada para penegak hukum hendaknya benar-benar harus tegas, adil, dan tidak pandang
bulu dalam memberantas korupsi di Indonesia.

10. Kalimat berikut yang merupakan kalimat baku adalah ...


A. Banyak para industriawan tidak dapat menyalurkan barang-barang produksinya karena
kurangnya dana untuk mengekspornya.
B. Pada masa sekarang, banyak orang-orang kebingungan mencari bahan-bahan kebutuhan
pokok karena harganya semakin melonjak drastis.
C. Di desa tertinggal di kaki gunung itu banyak anak menderita gizi buruk yang belum
tertangani secara intensif oleh tenaga medis
D. Seperti kita mengakui bahwa sejak Undang-Undang Pokok Agraria (UUPA) diberlakukan
masih banyak terjadi banyak ketimpangan-ketimpangan dari prinsip-prinsip UUPA.
E. Sekarang ini di ibu kota banyak perusahaan-perusahaan bergerak di bidang asuransi

11. Kalimat berikut yang merupakan kalimat tidak baku adalah ...
A. Di era globalisasi sekarang ini, sangatlah sulit jika kita tidak mengerti tentang bahasa lnggris.
B. Indonesia termasuk negara agraris yang sebagian besar penduduknya bercocok tanam.
C. Menurut para ekonom, dalam kondisi riil perekonomian Indonesia sekarang ini, asas
kekeluargaan dalam berbisnis tidak mudah diimplementasikan.
D. Seminar internasional yang diselenggarakan oleh pemerintah itu membahas bahasa, sastra,
dan budaya mutakhir.
E. Di antara tiga laptop di atas meja sekretaris, menurut Anda, mana yang terbaik?

12. Akibat vertical speed yang terlalu tinggi, pesawat menjadi sulit dikontrol saat roda hendak
menyentuh landasan sehingga benturan keras terjadi dan pesawat pun bisa terpental ke luar
Jandasan.

Kalimat inti dari kalimat di atas adalah ...

A. Vertical speed terlalu tinggi.


B. Pesawat sulit dikontrol.
C. Roda pesawat menyentuh landasan .
D. Terjadi benturan keras.
E. Pesawat terpental ke luar landasan.

13. Kalimat yang polanya tidak sama dengan kalimat yang lain adalah ...
A. Banyak buku dan tulisan mampu mengubah nasib sebuah bangsa.
B. Kegiatan membaca akan dapat membangun karakter masyarakat
C. Setiap pelajar diharuskan menghasllkan sebuah tulisan pendek.

3
alternatifa

D. Membaca tidak cukup dijadikan sebagai ajakan atau imbauan.


E. Mereka pasti akan menemukan buku yang tepat.

14. Dalam Konferensi lnternasional Kebudayaan Papua itu tidak diputuskan tempat penyelenggaraan
konferensi berikutnya.

Kalimat yang memiliki pola yang sama dengan kalimat di atas adalah ...

A. Untung Subagyo, pejabat yang terpilih itu, mengawali karirnya dari bawah.
B. Seperti telah kita ketahui, perekonomian Amerika Serikat sangat berpengaruh.
C. Telah kami bicarakan dalam rapat terdahulu bahwa kenaikan gaji pegawai baru bisa
dilaksanakan tahun depan.
D. Kita sadari bahwa bencana alam dapat datang kapan saja tanpa kita duga.
E. Berdasarkan pengarahan pimpinan, pembangunan kampus baru itu dapat dilakukan secara
bertahap.

15. Meskipun beberapa link yang dituliskan sudah berkali-kali dicermati, informasi yang
diperlukannya belum memadai.

Padanan kata yang tepat untuk kata link dalam kalimat di atas adalah …

A. Pranala
B. Saltik
C. Luring
D. Portofan
E. Warganet

16. lnsiden pesawat keluar landasan merupakan kecelakaan yang paling sering terjadi.

Kata yang tepat untuk menggantikan kata insiden dalam kalimat di atas adalah ....

A. Tindakan
B. Kejadian
C. Keadaan
D. Kecelakaan
E. Kenyataan

17. Kita tidak boleh berpangku tangan melihat dampak negatif media sosial.

Makna ungkapan berpangku tangan dalam kalimat di atas adalah …

A. Suka bermalas-malasan
B. Tidak berbuat apa-apa
C. Tidak bergeming
D. Tidak bekerja
E. Merenung saja

18. Pemerintah merasa bangga telah memprioritaskan hasil karya sendiri dalam menopang
pembangunan konstruksi nasional.

Kata yang tepat untuk menggantikan kata memprioritaskan dalam kalimat di atas adalah ....

4
alternatifa

A. Mengunggulkan
B. Mendahulukan
C. Memperhatikan
D. Mempertahankan
E. Menggunakan

19. Dalam memasuki era baru perpajakan, dibutuhkan perubahan paradigma seluruh elemen
bangsa, termasuk para pejabat negara, pegawai pajak, pegawai sektor keuangan, penegak hukum,
dan masyarakat wajib pajak. Sebagai inisiatif global, AEOI (Automatic Exchange of Information)
menuntut perubahan corak keterbukaan domestik agar penggalian potensi pajak lebih optimal
dan perbankan dalam negeri tidak justru menjadi tempat persembunyian baru bagi wajib pajak
dalam negeri. Era keterbukaan ini adalah cara efektif meningkatkan penerimaan pajak dengan
menegakkan prinsip ability to pay, yakni yang lebih mampu akan membayar lebih besar. Sistem
perpajakan harus mampu mendeteksi siapa memiliki apa (who own what) sebagai basis
pemajakan yang adil.

(Sumber: Koran Kompas, 7 Maret 2017)

Kesimpulan paragraf di atas adalah …

A. Perubahan paradigma perpajakan.


B. Perubahan corak keterbukaan.
C. Optimalisasi potensi pajak.
D. Efektivitas penerimaan.
E. Pemajakan yang berkeadilan.

20. Banyak orang sulit tersenyum. Mereka … bahwa hal--hal yang terburuk memang terjadi pada
dirinya. Mereka mempunyai kebiasaan melihat kepada orang-orang yang lebih beruntung dan
lupa melihat .... Orang-orang yang seperti ini kemudian terbiasa memfokuskan ... pada hal-hal
yang buruk semata. Begitu berita negatif datang, rangkaian pemikiran bernada suram terpicu
untuk …. Hal-hal yang mengalir ke luar bisa berbentuk gosip politik, kebencian, juga caci maki
terhadap golongan tertentu. Dampaknya adalah … berwarna abu-abu, dan tak pernah merah
muda.
(Sumber: Kompas, 18 Maret 2017)

Rumpang dalam paragraf di alas dapat dilengkapi dengan deret kata berikut ....

A. Marah, dirinya, kesukaannya, tercerahkan, hidup menyenangkan


B. Tidak percaya, hal-hal indah, dakwaannya, membabi buta, hidup suram
C. Beranggapan, ke bawah, perhatiannya, tercerahkan, hidup suram
D. Kurang yakin, hal-hal indah, perhatiannya, tercerahkan, hidup suram
E. Beranggapan, dirinya, tercerahkan, membabi buta, hidup suram

5
alternatifa

BAHASA INDONESIA UTUL UGM 2018 PAKET 585

Bacaan berikut ini dipergunakan untuk menjawab soal nomor 1-4

(1) Ketika kita membahas budaya, kita tidak hanya berbicara mengenai tari tradisional, pakaian adat,
maupun lagu daerah. (2) Namun, kita juga membahas budaya sebagai suatu cara hidup yang diwariskan
dari generasi ke generasi. (3) Budaya merupakan hal yang sangat penting bagi peradaban sebuah
bangsa karena budaya merupakan jati diri atau karakter yang mencerminkan bangsa tersebut. (4)
Sebuah bangsa tidak dapat dikatakan sebagai bangsa yang unggul apabila tidak memiliki budaya untuk
maju. (5) Sementara itu, menciptakan budaya maju tidak seperti membalikkan telapak tangan. (6) Perlu
ada "cambuk" dari dalam diri bangsa itu sendiri untuk bisa menjadi bangsa yang memiliki budaya maju.

(Sumber: http://www.kompasiana.com/hagungps/20 April 2018)

1. Inti bacaan di atas adalah ...


A. Budaya maju penting bagi suatu bangsa.
B. Budaya penting bagi peradaban sebuah bangsa.
C. Budaya merupakan pandangan hidup suatu bangsa.
D. Pembicaraan tentang budaya bangsa berhubungan dengan cara hidup.
E. Penciptaan budaya untuk maju perlu "cambuk".

2. Gagasan utama bacaan di atas adalah ....


A. Kerja keras menciptakan budaya maju.
B. Pentingnya budaya maju untuk suatu bangsa.
C. Budaya sebagai peradaban sebuah bangsa.
D. Budaya jati diri sebagai peradaban sebuah bangsa.
E. Cara-cara penciptaan budaya maju bagi sebuah bangsa.

3. Ringkasan bacaan di atas adalah ...


A. Budaya sebagai jati diri bangsa perlu dipelihara dan diwariskan dari generasi ke generasi.
Bila tidak, bangsa tersebut akan kehilangan identitasnya dan tidak akan dapat mencapai
kemajuan.
B. Membahas budaya suatu bangsa tidak hanya berbicara mengenai tari tradisional, pakaian
adat, dan lagu daerah. Namun, juga membahas suatu cara hidup yang diwariskan dari
generasi ke generasi
C. Budaya untuk maju merupakan hal yang sangat penting bagi peradaban sebuah bangsa,
bahkan menentukan unggul tidaknya bangsa tersebut. Meskipun tidak mudah, bangsa
tersebut perlu berusaha keras untuk mencapainya.
D. Budaya untuk maju merupakan hal yang sangat penting bagi peradaban sebuah bangsa. Oleh
karena itu, setiap bangsa perlu membina rakyatnya agar menyadari dan mampu
mengimplementasikan dalam kehidupan sehari-hari.
E. Karakter bangsa tercermin dalam budayanya. Oleh karena itu, memiliki budaya maju dapat
dilihat dari upaya yang dilakukan bangsa tersebut.

4. Pernyataan yang tidak sesuai dengan isi bacaan di atas adalah ...
A. Budaya merupakan jati diri dari sebuah bangsa.
B. Bangsa yang unggul adalah bangsa yang memiliki budaya maju.

1
alternatifa

C. Menciptakan budaya maju tidaklah mudah.


D. Bangsa yang unggul belum tentu memiliki budaya maju.
E. Budaya dapat pula berarti cara hidup yang diwariskan dari generasi ke generasi.

Bacaan berikut ini dipergunakan untuk menjawab soal nomor 5- 8

(1) Hingga tahun 2017, pertumbuhan ekonomi Indonesia berada di kisaran 5 persen. (2) Pada tahun ini
pun, pertumbuhan ekonomi Indonesia ditargetkan berada pada kisaran 5,1 hingga 5,4 persen. (3) Tren
pertumbuhan ekonomi Indonesia sudah ada perbaikan. (4) Ini terutama berasal dari investasi dan
perdagangan. (5) Pertumbuhan ekonomi Indonesia bisa mencapai kisaran 6-7 persen. (6) … ada syarat
yang harus dipenuhi, yakni struktur ekonomi Indonesia harus diubah. (7) Pendorong pertumbuhan
ekonomi saat itu ialah konsumsi rumah tangga dan investasi. (8) Agar pertumbuhan ekonomi Indonesia
dapat terakselerasi, infrastruktur harus digenjot. (9) Namun demikian, ini tidak bisa terjadi dalam
jangka pendek, tapi jangka menengah dan panjang sejalan pembangunan infrastruktur.

(sumber: https/ekonomi.kompas.com)

5. Isian yang tepat untuk melengkapi bagian rumpang dalam bacaan di atas adalah …
A. Sedangkan
B. Akan tetapi
C. Selanjutnya
D. Meskipun
E. Bahkan

6. Kalimat yang tidak mendukung isi bacaan di atas adalah …


A. kalimat (3)
B. kalimat (6)
C. kalimat (7)
D. kalimat (8)
E. kalimat (9)

7. Kalimat yang tidak mendukung isi bacaan di atas adalah …


A. kalimat (4)
B. kalimat (5)
C. kalimat (6)
D. kalimat (7)
E. kalimat (8)

8. Inti kalimat (8) dalam bacaan di atas adalah ...


A. Pertumbuhan ekonomi Indonesia terakselerasi.
B. Infrastruktur digenjot.
C. Pertumbuhan ekonomi harus digenjot.
D. Pertumbuhan ekonomi Indonesia jelas.
E. Regulasi dan kepastian hukum jelas.

Bacaan berikut ini dipergunakan untuk menjawab soal nomor 9- 12

2
alternatifa

(1) Dalam skema yang telah berjalan tiga tahun, institusi Jepang dan Indonesia berinvestasi
dalam kegiatan pembangunan rendah karbon di Indonesia melalui insentif dari Pemerintah Jepang. (2)
Indonesia menyadari, pembangunan rendah karbon sebagai sebuah tren baru. (3) Diupayakan untuk
menggenjot ekonomi tidak boleh dipisahkan dari pertumbuhan rendah emisi dan pertumbuhan
berkelanjutan. (4) Seluruh implementasi dalam kerangka JCM (Joint Crediting Mechanism) seperti
efisiensi energi, pembangkit listrik energi terbarukan, manajemen limbah transportasi, dan manajemen
lahan harus mengkualifikasi kegiatan rendah karbon atau mampu mengurangi emisi CO2, sembari
menjaga produktivitas dan efisiensi. (5) Selama tiga tahun berlangsungnya kerja sama bilateral, skema
JCM berhasil menyalurkan lebih dari US$37 juta subsidi kepada pihak swasta Indonesia. (6) Kontribusi
pihak swasta Indonesia dalam skema ini mencapai US$113 juta, sehingga total nilai investasi dalam
skema JCM telah mencapai lebih dari US$150 juta. (sumber: https://www.cnnnidoinesia.com)

9. Pemakaian kata yang tidak tepat terdapat pada …


A. kalimat (1)
B. kalimat (2)
C. kalimat (3)
D. kalimat (4)
E. kalimat (5)

10. Kata bentukan yang salah terdapat pada kalimat ....


A. kalimat (1)
B. kalimat (2)
C. kalimat (3)
D. kalimat (4)
E. kalimat (5)

11. Hubungan antara kalimat (2) dan kalimat (3) pada bacaan di atas dapat diperjelas dengan kata
penghubung …
A. ketika itu
B. di samping itu
C. karena itu
D. selain itu
E. selanjutnya

12. Padanan kata serapan efisiensi yang tepat dalam bacaan di atas adalah ....
A. ketepatan cara
B. berdaya guna
C. bertepat guna
D. sesuai guna
E. berhasil guna

Bacaan berikut ini dipergunakan untuk menjawab soal nomor 13- 16

(1) Kementerian Riset, Teknologi, dan Pendidikan Tinggi (Kemenristekdikti) dalam tiga tahun terakhir
terus-menerus menggenjot peningkatan jumlah publikasi ilmiah Indonesia. (2) Hal itu dilakukan
melalui berbagai kebijakan demi mendorong para Profesor, dosen, dan peneliti untuk produktif menulis
publikasi ilmiah. (3) Publikasi bukan hanya sebagai tugas dan kewajiban semata, melainkan juga

3
alternatifa

menjadi tolak ukur keberhasilan sebuah riset karena kemajuan suatu bangsa dapat dilihat dari riset
maupun pengembangan ilmu pengetahuan dan teknologi. (4) Setelah melampaui Thailand sampai akhir
tahun 2017 dengan jumlah publikasi ilmiah internasional Indonesia mencapai angka 18 500, pada tri
wulan pertama tahun 2018 Indonesia berhasil menggeser Singapura, sehingga menempati urutan ke-2
di ASEAN setelah Malaysia. (5) Berdasarkan data dari Direktorat Jenderal Penguatan Riset dan
Pengembangan, jumlah publikasi ilmiah Indonesia terindek Scopus per 6 April 2018 berhasil melampaui
Singapura dan Thailand.

(Dikutip dengan perubahan dari Kompas.com, 12 April 2018)

13. Pemakaian huruf kapital yang salah terdapat dalam kalimat …


A. kalimat (1)
B. kalimat (2)
C. kalimat (3)
D. kalimat (4)
E. kalimat (5)
14. Penulisan kata yang salah terdapat dalam kalimat …
A. (1)
B. (2)
C. (3)
D. (4)
E. (5)

15. Penulisan unsur serapan yang salah terdapat dalam …


A. kalimat (1)
B. kalimat (2)
C. kalimat (3)
D. kalimat (4)
E. kalimat (5)

16. Penulisan tanda baca koma (,) yang salah terdapat dalam …
A. kalimat (1)
B. kalimat (2)
C. kalimat (3)
D. kalimat (4)
E. kalimat (5)

(1) Pengangguran sering kali dijumpai di sekeliling kita. (2) Banyak orang menyebutkan
sebagai tuna karya. (3) Dalam hal yang demikian, pengangguran dapat berarti orang yang sedang tidak
bekerja apa pun dengan alasan tertentu. atau seseorang yang sedang mencari kerja, dan dapat pula
berarti orang yang bekerja kurang dari waktu normal orang bekerja. (4) Pengangguran pada umumnya
dapat terjadi karena jumlah angkatan kerja yang tidak sebanding dengan jumlah lapangan kerja yang
disediakan. (5) Pengangguran sering kali menjadi sebuah masalah yang menimpa seseorang. (6)
Pengangguran ini juga dapat berdampak kepada perekonomian suatu negara yang disebabkan karena
tingginya tingkat pengangguran. (7) Dampaknya akan membuat produktivitas dan pendapatan
masyarakat serta negara akan berkurang sehingga jika terjadi dalam jangka panjang dapat
menyebabkan timbulnya kemiskinan serta dapat pula menyebabkan masalah-masalah sosial lainnya.

4
alternatifa

(sumber: http:/www.informasi-pendidikan.com)

17. Kalimat (6) dengan kalimat (7) dalam bacaan di atas mempunyai hubungan makna …
A. penjelasan
B. perturutan
C. sebab-akibat
D. kegunaan
E. contoh

18. Kalimat (5) dengan kalimat (6) dalam bacaan di atas dapat dijadikan satu kalimat dengan kata
penghubung …
A. lalu
B. bahkan
C. sedangkan
D. sementara
E. padahal

19. Perbaikan kalimat yang tepat untuk kalimat (6) dalam bacaan di atas adalah ...
A. Pengangguran juga dapat berdampak kepada perekonomian suatu negara yang disebabkan
karena tingginya tingkat pengangguran.
B. Pengangguran juga dapat berdampak kepada perekonomian suatu negara karena tingginya
tingkat pengangguran.
C. Pengangguran dapat berdampak kepada perekonomian suatu negara yang disebabkan
tingginya tingkat pengangguran.
D. Pengangguran ini juga dapat berdampak kepada perekonomian suatu negara disebabkan
karena tingginya tingkat pengangguran.
E. Pengangguran dapat berdampak pada perekonomian suatu negara karena tingginya tingkat
pengangguran.

20. Gabungan yang tepat untuk kalimat (1) dengan kalimat (2) dalam bacaan di atas adalah ...
A. Pengangguran sering kali dijumpai di sekeliling kita dan banyak orang menyebutnya sebagai
tuna karya.
B. Pengangguran sering kali dijumpai di sekeliling kita, jadi banyak orang menyebutnya
sebagai tuna karya.
C. Pengangguran sering kali dijumpai di sekeliling kita, malah banyak orang menyebutnya
sebagai tuna karya.
D. Pengangguran sering kali dijumpai di sekeling kita, apalagi banyak orang menyebutnya
sebagai tuna karya.
E. Pengangguran sering kali dijumpai di sekeliling kita, bahkan banyak orang menyebutnya
sebagai tuna karya.

5
alternatifa

UTUL UGM BAHASA INDONESIA 2019 634

Bacaan berikut ini dipergunakan untuk menjawab soal nomor 1-4

(1) Selain di kenal sebagai Pulau Cinta atau Pulau Dewata, Bali juga ingin menyandang predikat sebagai
Pulau Energi Bersih. (2) Program Bali sebagai percontohan pengembangan energi bersih ini, sudah
digaungkan sejak tahun 2015 lalu. (3) Bali memiliki modal yang kuat untuk berhasil karena upaya untuk
mengembangkan energi terbarukan selaras dengan Filosofi trihita karana yang melekat di dalam
budaya masyarakat Bali. (4) Trihita karana mengandung makna menjaga keseimbangan hubungan
tiga elemen antara manusia dengan Tuhan, sesama manusia, dan lingkungan. (5) Perayaan Nyepi yang
senyap menjadi puncak sekaligus acuan yang kuat yang menunjukkan harmonis dari trihita karana
tersebut. (6) Saat perayaan Nyepi, selama 24 Jam masyarakat Bali hanya beraktivitas di dalam rumah,
tidak bepergian. (Sumber: kompas.id, 7 Maret 2019)

1. Pemakaian huruf kapital yang salah 3. Pemakaian kata serapan yang salah
terdapat dalam kalimat .... terdapat dalam kalimat …
A. (1) A. (1)
B. (2) B. (3)
C. (3) C. (4)
D. (5) D. (5)
E. (6) E. (6)

2. Dalam bacaan di atas terdapat penulisan 4. Pemakaian tanda baca koma (,) yang tidak
kata yang tidak sesuai dengan pedoman tepat terdapat dalam kalimat …
ejaan bahasa Indonesia, yaitu .... A. (2)
A. di kenal B. (3)
B. trihita karana C. (4)
C. 24 jam D. (5)
D. menunjukkan E. (6)
E. bepergian

Bacaan berikut ini dipergunakan untuk menjawab soal nomor 5-9.

(1) Kementerian Pertanian berkomitmen untuk terus meningkatkan pengembangan buah-buahan lokal
asli tanah air. (2) Salah satunya adalah buah durian. (3) Salah satu petani yang mengelola kebun 40 hektare
dengan 4.000 pohon, yakni Anjung. (4) Dia membudidaya jenis durian matahari yang pada saat panen raya
mampu berbuah paling banyak 1.000 butir per pohon. (5) Harga durian menurutnya masih murah di
petani, yakni Rp15.000,00 per butir. (6) "Harga di petani ini berbeda dengan di Jakarta yang bisa mencapai
kisaran Rp40.000,00 hingga Rp50.000,00 per butir,” ujarnya. (7) Menurut Anjung, perbedaan harga
tersebut justru merupakan sebuah peluang bisnis yang cukup terbuka. (8) Dia memiliki durian lokal yang
unik, belum ada di daerah lain, yakni jenis madu dan racun yang bisa berbuah hingga mencapai berat 8
kilogram per butir.

(Sumber : https:ekbis.sindonews.com/read; 2 Maret 2019)

5. Pilihan kata yang tidak tepat terdapat A. (1)


dalam kalimat …. B. (2)

1
alternatifa
C. (3) D. Saat ini,
D. (4) E. Namun,
E. (5)
8. Padanan kata yang tepat untuk kata
6. Bentukan kata yang tidak tepat terdapat serapan bisnis adalah .. ..
dalam kalimat .... A. usaha
A. (1) B. perdagangan
B. (2) C. dagang
C. (3) D. kegiatan
D. (4) E. urusan
E. (5)
9. Pemakaian kata tidak baku terdapat
7. Hubungan antara kalimat (4) dan (5) dalam kalimat…
dapat dipertegas dengan kata penghubung A. (2)
... B. (3)
A. Dengan demikian, C. (4)
B. Karena itu, D. (5)
C. Bahkan, E. (6)

Bacaan berikut ini dipergunakan untuk menjawab soal nomor 10-15

(1) Keberadaan bank sampah mulai menjadi tren di Kota Serang, Banten. (2) Warga diajak menekan
masalah sampah dengan proses pemilahan dari rumah. (3) ... (4) Oleh karena itu, di sana kemudian
terbentuklah Bank Sampah Barokah pada pertengahan tahun 2018. (5) Keberadaannya berawal darl
imbauan Dinas Lingkungan Hidup Kota Serang yang direalisasikan para aparatur-aparatur Kelurahan
Cipete. (6) Tujuannya adalah untuk mengurangi beban tempat penampungan sementara dan Tempat
Pembuangan Akhir Cilowong di Kota Serang mulai dari rumah warga.

(7) Kini ada 12 bank sampah. (8) Bank itu terbentuk di 11 kelurahan di 4 kecamatan. (9) Pada tahun 2020,
Dinas Lingkungan Hidup Kota Serang menginginkan bank sampah sudah terbentuk di semua kelurahan,
yaitu 67 kelurahan. (10) Sembari menunggu warga lain bergerak, benih kepedulian tumbuh subur di Bank
Sampah Barokah. (11) Nasabah di bank sampah ini untuk sementara ini berjumlah 15 orang yang dilayani
15 pengurus.

(Sumber: kompas.id, 6 Maret 2019)

10. Isian yang tepat untuk bagian rumpang A. Keberadaannya berawal daripada
dalam bacaan di atas adalah ... imbauan Dinas Lingkungan Hidup
A. Selanjutnya, limbah disalurkan Kota Serang yang direalisasikan para
kepada pengepul barang rongsokan. aparatur Kelurahan Cipete.
B. Harapannya, sampah berkurang, B. Keberadaannya berawal dari
rupiah pun datang. imbauan Dinas Lingkungan Hidup
C. Setelah setor sampah, nasabah tak Kota Serang yang direalisasikan para
langsung bayar. aparatur-aparatur Kelurahan Cipete.
D. Masyarakat yakin bank sampah bisa C. Keberadaannya mulai berawal
membantu memperoleh penghasilan. daripada imbauan Dinas Lingkungan
E. Setiap kali menyetor limbah, jumlah Hidup Kota Serang yang
uang yang diterima bervariasi. direalisasikan para aparatur
Kelurahan Cipete.
11. Perbaikan yang paling tepat untuk kalimat D. Keberadaannya berawal dari
(5) adalah … imbauan Dinas Lingkungan Hidup

2
alternatifa
Kota Serang yang direalisasikan para D. Tujuannya adalah Tempat
aparatur Kelurahan Cipete. Pembuangan Akhir
E. Keberadaannya berawal dari E. Tujuannya adalah mengurangi
imbauan Dinas Lingkungan Hidup rumah warga.
Kota Serang yang direalisir para
aparatur Kelurahan Cipete. 14. Apa simpulan bacaan di atas?
A. Sampah merupakan barang bekas.
12. Gabungan yang paling tepat untuk kalimat B. Sampah dapat didaur ulang kembali.
(7) dan kalimat (8) adalah … C. Bank sampah mengatasi masalah
A. Kini ada 12 bank sampah karena sampah.
terbentuk di 11 kelurahan di empat D. Bank sampah sudah diberdayakan.
kecamatan. E. Bank sampah sangat membantu
B. Kini ada 12 bank sampah serta warga.
terbentuk di 11 kelurahan di empat
kecamatan. 15. Kalimat yang paling tepat untuk
C. Kini ada 12 bank sampah sementara melengkapi informasi dalam paragraf
terbentuk di 11 kelurahan di empat kedua adalah ...
kecamatan. A. Pengurus bank sampah biasanya
D. Kini ada 12 bank sampah yang mendatangi setiap nasabah untuk
terbentuk di 11 kelurahan di empat memudahkan sistem pembelian
kecamatan. sampah.
E. Kini ada 12 bank sampah, lalu B. Jenis sampah dan harga yang
terbentuk di 11 kelurahan di empat diberikan oleh bank sampah ini
kecamatan. cukup bervariasi.
C. Itulah sebabnya dinamakan bank
13. Inti kalimat (6) adalah ... karena punya nasabah dan buku
A. Tujuannya adalah mengurangi tabungan.
beban. D. Harapannya, uang yang diserahkan
B. Tujuannya adalah mengurangi lebih banyak dan tidak langsung
tempat penampungan. habis saat itu juga.
C. Tujuannya adalah sementara. E. Hanya koran bekas yang disisihkan
untuk diolah menjadi kerajinan.

Bacaan berikut ini dipergunakan untuk menjawab soal nomor 16-20.

(1) Kampung Bahasa atau yang lebih dijuluki Kampung lnggris di Pare yang berjarak sekitar 25 kilometer
dari pusat kota Kediri merupakan pusat pelatihan bahasa Inggris yang mulai berdiri pada tahun 1977 dan
memiliki ratusan lembaga kursus pengajaran bahasa. (2) Sebutan Kampung lnggris kemudian berubah
menjadi Kampung Bahasa karena jenis bahasa yang diajarkan terus bertambah, seperti bahasa Arab,
Jerman, Prancis, Korea, Jepang, dan Mandarin.

(3) Sejumlah lembaga kursus di Kampung Bahasa pun menawarkan jenis program pelatihan yang berbeda-
beda. (4) BEC (Basic English Course) lembaga tertua dan pertama di Pare, hanya menawarkan program
enam bulan dan tidak mengelola tempat menginap (homestay). Lembaga lain, Genta, menawarkan program
maksimal dua bulan hingga satu tahun dan wajib tinggal di asrama. (6) Bahkan, sejumlah lembaga
melengkapi program keterampilan mengajar dan kewirausahaan. (7) Kampung Bahasa sudah dikenal luas
oleh masyarakat yang ingin belajar bahasa asing.

(sumber: https/bebas.kompas.id/baca/utama; 6 Maret 2019)

16. Gagasan utama bacaan di atas adalah .... B. kursus bahasa di Kampung Bahasa
A. Kampung Bahasa di Pare C. berbagai lembaga kursus bahasa

3
alternatifa
D. sebutan Kampung lnggris
E. belajar bahasa asing di Pare

17. Kalimat utama dalam paragraf kedua


terdapat pada kalimat …
A. (1)
B. (2)
C. (3)
D. (4)
E. (5)

18. Kalimat yang tidak berhubungan dengan


isi bacaan di atas adalah kalimat …
A. (3)
B. (4)
C. (5)
D. (6)
E. (7)

19. Masalah yang dibahas dalam bacaan di


atas adalah …
A. Perubahan nama Kampung lnggris
menjadi Kampung Bahasa
B. Kampung Bahasa dan program
pelatihan yang ditawarkan
C. Berbagai program pelatihan bahasa
asing
D. Kampung Bahasa dan pengajaran
bahasa asing
E. Letak Kampung Bahasa dan program
pelatihan bahasa.

20. Inti bacaan di atas adalah …


A. Kampung Bahasa merupakan nama
baru dari Kampung lnggris.
B. Sejumlah lembaga kursus ada di
Kampung Bahasa.
C. Kampung Bahasa memiliki berbagai
program pelatihan
D. Kampung lnggris mengajarkan
berbagai jenis bahasa.
E. Lembaga kursus di Kampung Bahasa
memiliki berbagai program
pelatihan.

Anda mungkin juga menyukai